CIVPRO 1 / II MCQ

Réussis tes devoirs et examens dès maintenant avec Quizwiz!

Question 4 6 / 6 pts In a products liability case, one defendant from Nevada marketed its product to the state of California, and designed the product for consumers within the state, where the product injured a consumer. Can the California court exercise jurisdiction over the defendant from Nevada? Yes, because the stream of commerce test under Asahi is satisfied. Yes, only if the defendant itself is physically present through retail operations in the state. No, because the court's exercise of jurisdiction would not be consistent with traditional notions of fair play and substantial justice. No, because the defendant is not considered present in the forum.

A is the best answer under these facts, since --both the O'Connor and Brennan stream of commerce requirements are met. B is incorrect, because --retail operations in the state --would not be required for personal jurisdiction. --C is incorrect because traditional notions would be satisfied under the facts, --based on the Asahi case. D is incorrect because --even if defendant is not present in the forum, personal jurisdiction is available on these facts.

Question 5 0 / 10 pts Plaintiff Tenant asserts a claim against defendant Landlord for violation of the federal civil rights laws prohibiting racial discrimination in housing. The statute provides for a private right of action but is silent regarding any right to a jury trial. The Plaintiff seeks compensatory and punitive damages. Defendant requests a jury trial on the claim. Which factors suggest that a right to jury trial should be allowed? I. The rights are like common law torts. II. The remedy is a legal remedy. III. The statutes did not exist historically. All of the above. I and II only. II and III only. I and III only.

B is correct based on --Curtiss v. Loether in which --the court allowed a jury trial, --not because --the right existed historically as a legal action, but because --the right is a tort-like right, and --the remedy a legal one. A, C and D are incorrect because --Chauffers allows jury trial for --new statutory rights, --not just those that existed historically. Quiz Score: 30 out of 50

Question 9 4.5 / 4.5 pts Paul, a resident of Oregon, filed a claim for personal injuries against Dell, a California resident, who ran into Paul's car when Dell was driving a delivery truck for Dan's Deli, a Delaware Corporation with its principal place of business in Dover, Delaware. At the same time, Paul filed a claim against Dan's Deli, based on vicarious liability for Paul's negligent conduct. Dan's Deli brought a third party claim against Dell and a claim against Zorro Insurance, for indemnity. Which of the following are correct? The claim brought by Dan's Deli against Dell is a counterclaim and its claim against Zorro Insurance is an impleader claim. The claim brought by Dan's Deli against Dell is a cross-claim and its claim against Zorro Insurance is an impleader claim. The claim brought by Dan's Deli against Dell is a cross-claim and its claim against Zorro Insurance is an interpleader claim. The claim brought by Dan's Deli against Dell is a counterclaim and its claim against Zorro Insurance is a claim in intervention.

B is correct because a claim against a co-party is a cross-claim and a claim against a third party is an impleader claim. A is incorrect because a counterclaim is a claim against an opposing party. C is incorrect because a claim against a third party is an impleader claim, not an interpleader claim. D is incorrect because the claim against Zorro is an impleader claim, not a claim in intervention.

Question 2 0 / 6 pts Harvey's filed suit against Blitter, seeking payment of $80,000 based on promissory notes executed by Blitter. Harvey's is a Nevada corporation and Blitter is a California citizen. Harvey's failed to bring the case to trial within five years as required by a Nevada State Civil Procedure statute and Blitter moved to dismiss the case. Rule 41(b) of the Federal Rules of Civil Procedure permits dismissal for failure to prosecute a claim. The Nevada statute mandates dismissal for failure to prosecute a claim. Should a federal court in Nevada apply the Nevada rule or Federal Rule of Civil Procedure 41? To avoid forum shopping, the state rule should apply. The Federal Rule of Civil Procedure should apply, because it is on point and valid. State law should apply because the Federal Rule of Civil Procedure is not on point. None of the above.

B is the best answer - the court --will apply the FRCP, --based on the Hanna analysis: --the rule is on point, and --it is valid, so it should be applied unless --it abridges a substantive right. --Courts have never found any rule to do so, and --the rules are presumptively valid. In interpreting Rule 41(b), the rule applies because --it mandates exercise of discretion and --can be read to prohibit automatic dismissals. A is incorrect because --it is unlikely that any one filing a suit would --choose a forum based on --whether dismissal for failure to prosecute --is mandatory or permissive, so --this difference would not lead to forum shopping. C is incorrect because --the FRCP is on point, since --it in fact addresses whether a case should be --dismissed based on a failure to prosecute the case.

Question 3 6 / 6 pts Jock, a sports executive and resident of Houston (located in the Southern District of Texas) visited New Orleans on a scouting trip. Jock filed an assault claim against Nell, a New Orleans resident (in the Eastern District of Louisiana) and her boyfriend Bill, a resident of Baton Rouge (located in the Middle District of Louisiana) after Bill punched Jock, during a brawl at a sporting event in New Orleans. Jock also sued the Silver Arena, where the events occurred for failing to provide adequate security. The Arena is a Delaware corporation. Where is venue proper in Jock's case? Southern District of Houston Either district in Louisiana. The Middle District of Louisiana only. The District of Delaware only.

B is the best answer because --the districts in Louisiana are --where the defendants reside and --where the events occurred A is incorrect because --the plaintiff's location is not a proper venue. C is incorrect because --the Eastern District of Louisiana is also a proper venue. D is incorrect because --venue would also be proper in Louisiana.

Question 3 10 / 10 pts Marty sued Felix for breach of contract. Felix moved to dismiss for lack of subject matter jurisdiction. The motion was denied. Felix then filed an answer in which he raised the defense of lack of personal jurisdiction and failure to state a claim. Did Felix waive either of the two defenses? Both were waived. Only the defense based on lack of personal jurisdiction was waived. Only the defense based on failure to state a claim was waived. Neither defense was waived.

B is the best answer under Rule 4 because --the personal jurisdiction defense is --waived if not raised in Felix's initial response, but --Felix remains free to assert failure to state a claim. Therefore --A, C and D are incorrect.

Question 5 6 / 6 pts A State A shopkeeper filed a complaint against a State B supplier in State A federal court based on the supplier's failure to deliver goods to the shopkeeper in a timely manner. The shopkeeper is seeking $100,000 in damages. After receiving the shopkeeper's complaint, the supplier filed a third party complaint against the delivery company, a State B corporation headquartered there, alleging that the delivery company is partially responsible for the delay and thus should be liable for 50% of any liability the supplier is found to have to the shopkeeper. Assume in State A that joint tortfeasors are subject to contribution claims. Does the court have subject matter jurisdiction over the supplier's claim against the delivery company? Yes, the court has original jurisdiction over the supplier's claim against the delivery company. Yes, because the claim involves the same case or controversy as the shopkeeper's claim against the supplier No, because the claim does not arise from the same facts. No, because the claim is not asserted by the shopkeeper.

B is the best answer, because --Section 1367 extends supplemental jurisdiction to --claims that are so related to the claims in the action within the court's original jurisdiction that --they form part of the same case or controversy, and --the supplier's claim involves the same delivery as the shopkeeper's claim. A is incorrect because --the court does not have original jurisdiction over the supplier's claim due to --the amount in controversy being $50,000, which is --under the $75,000 plus required. C is incorrect because --the claim does arise from the same facts. D is incorrect because --whether the claim is asserted by the shopkeeper --does not alone determine whether --there is supplemental jurisdiction, and --claims by plaintiffs (here the shopkeeper) --are more likely to be excluded under 1367 (b). Quiz Score: 12 out of 30

Question 5 10 / 10 pts Pam's husband died in a plane crash. Ten survivors filed separate wrongful death cases against the Airline in ten separate states. In seven cases, the jury found that the Airline was not negligent. In three cases, the jury found that the Airline was negligent. Pam filed case number 11 in her own state, seeking damages for wrongful death against Airline. Pam files a motion for summary judgment, seeking to use the determinations that the Airline was negligent in three cases to preclude the Airline from litigating the cases based on the doctrine of res judicata/claim preclusion. How should the court rule? Deny the motion because it is not fair for Pam adopted a wait and see attitude. Deny the motion because the parties differ and Pam was not in privity with the other plaintiffs. Grant the motion because there was a final valid judgment on the merits. Grant the motion because the claims involve the same transaction or occurrence.

B is the best answer, because --res judicata requires --the same parties or --that parties be in privity. --Privity is not present here, so --the doctrine will not apply to --preclude Airline from re-litigating the claim. A is incorrect because --there is no indication Pam did so. C is incorrect because --this is only one requirement for res judicata to apply, and --the doctrine will not apply where --the other requirements are not met. D is incorrect for the same reason as C. Quiz Score: 40 out of 50

Score for this quiz: 24 out of 30 Submitted Jul 25 at 11:24pm This attempt took 14 minutes. Question 1 0 / 6 pts In which of the following does a federal court have subject matter jurisdiction based on a federal question? I. Plaintiff sues for defamation, asserting that assertion of the defense of privilege would violate the US Constitution. Plaintiff and defendant are from the same state. II. Plaintiff files a claim for patent infringement. Both I and II. I only. II only None of the above

C is the best answer - since --the patent infringement claim --raises a federal question based on --a federal statute. I does not raise a federal question based on the well pleaded complaint rule. Therefore A and B are incorrect. --D is incorrect because --II is a basis for --federal question subject matter jurisdiction.

Question 6 4 / 4 pts Plaintiff Paula, a resident of New York, sued defendant Dave, a resident of New Jersey, in a New Jersey state court, for $100,000 based on breach of contract. Which of the following are true? Paula can remove the case to federal court, because the court has subject matter jurisdiction over the case. Dave can remove the case to federal court because the court has subject matter jurisdiction over the case. Dave cannot remove the case to federal court even though the case could have been filed in federal court. Dave can remove the case to federal court only if Paula agrees to do so.

C is correct because --an in-state defendant --cannot remove a case to federal court --even if the case could have originally --been filed in federal court. --In diversity cases, the action can be removed only if --the defendant is not a citizen of the state where the matter is pending. A is incorrect because --plaintiffs cannot remove cases to federal court --under any circumstances - --only a defendant can remove. B is incorrect because --an in state resident cannot remove a case to federal court. D is incorrect because --even if Pamela agrees, --Dave cannot remove the case, given that --he is an in state defendant.

Question 3 6 / 6 pts Plaintiff alleges a claim that state administrative procedures violate the due process and equal protection clauses of the US Constitution. Defendant moves to dismiss the claim, asserting that the claim does not assert a federal question. Assume the parties are not diverse and that the amount in controversy is $80,000. How should the court rule on the motion? Grant the motion, because the federal question is not substantial. Grant the motion because there is no subject matter jurisdiction based on diversity. here is subject matter jurisdiction Deny the motion, because the federal claim is substantial. Deny the motion, because the court has subject matter jurisdiction based on diversity.

C is the best answer - deny the motion because --there is a substantial federal question. Even though the claim is a state claim, --it raises federal constitutional questions. This is similar to the City of Chicago v. Int'l College of Surgeons case and the closest case is Smith. A constitutional question is very significant and --essential to this claim (like Grable). In addition, the --structural consequences will be very low. Because of this, A is incorrect. B is incorrect because --even though there is no subject matter jurisdiction based on diversity, --there is subject matter jurisdiction --based on a federal question, so --the motion to dismiss should not be granted. D is incorrect because --the parties are not diverse, so --there is no subject matter jurisdiction based on diversity.

Question 4 6 / 6 pts Cola Co. is a bottler of soft drinks, located in the state of New York, whose products are distributed throughout the United States. Pam is a resident of Illinois who was injured when a soda bottle made by Cola Co. exploded. Cola Co. ships a large quantity of product into the state of Illinois and also advertises extensively in the state. Pam sued Cola Co. in the State of Illinois. The long arm statute permits service on out-of-state defendants who commit tortious conduct in the state of Illinois. Cola Co. moved to dismiss the case due to lack of personal jurisdiction. The court should: Grant the motion, based on an absence of minimum contacts. Grant the motion, based on the absence of purposeful availment. Deny the motion only if Cola Co. owns property within the state of Illinois. Deny the motion, because the court has specific jurisdiction over Cola Co. in Illinois.

Correct! D is the best answer because --Cola Co has shipped its product into Illinois and advertised in the state, which will suffice under Asahi, and the tortious injury occurred in the state. A is incorrect because --there is a minimum contact with the state based on the shipment of products and advertising. B is incorrect because --there is purposeful availment based on shipment of product into the state as well as advertising. C is incorrect, because -even if Cola Co. does not own any property within the state, there is still personal jurisdiction based on a stream of commerce theory.

Question 3 6 / 6 pts Preston, a California citizen, brought a personal injury suit against Delia, a New York citizen, in federal district court in California arising out of an auto accident that occurred in New York City. Last year, Delia inherited from her Aunt Mabel a home in Irvine, California which she leased to tenants. Delia's Answer denied that she was negligent. After a trial, the jury awarded Preston a verdict in the amount of $78,000. On appeal from the judgment entered on the verdict, Delia seeks a reversal of the judgment on the grounds that the federal court in California did not have personal jurisdiction over her and, therefore, the judgment is invalid and unenforceable. In ruling on Delia's appeal, the Court of Appeal should: reverse the judgment because Delia's contacts with California did not give rise to Preston's cause of action. reverse the judgment because Delia's contacts with California were not purposeful. affirm the judgment because Delia's home in Irvine supports general jurisdiction. affirm the judgment because Delia waived her personal jurisdiction defense.

D is the best answer because --Delia did not raise the issue of personal jurisdiction, and --failure to raise the issue waives the issue, so that --she cannot raise it on appeal. A, B and C are incorrect because --Delia has waived the defense.

Question 13 4 / 4 pts Plaintiff Paulo, a resident of Mexico, sells goods manufactured in Mexico to defendant Dillon, a resident of Texas. Dillon failed to pay Paulo for a shipment and returned another shipment in damaged condition. Paulo sued Dillon in federal district court in Texas, alleging two claims: (1) a claim for $20,000 for breach of contract; and (2) a claim for $60,000 for goods that were returned damaged. Does the court have subject matter jurisdiction? No, because the parties are not diverse since Paulo is doing business in Texas. No, because the amount in controversy is not satisfied by either (1) or (2). Yes, but only if the case involves a federal statute. Yes, because Paulo can aggregate his claims to meet the amount in controversy.

D is the best answer here because --the parties are diverse based on --Paulo's alienage given his domicile in Mexico and --Paulo can aggregate his claims against a single defendant to meet the amount in controversy. A is incorrect because --the parties are diverse. B is incorrect because --the amount in controversy is satisfied since Paulo can aggregate the $20,000 and $60,000 claims. C is incorrect because --there is jurisdiction here based on diversity given that aggregation of amounts in controversy by one plaintiff against one defendant is permissible.

Question 2 6 / 6 pts Under which of the following scenarios does the court have subject matter jurisdiction, assuming the parties are diverse? I. Plaintiff alleges $75,000 in damages for breach of contract. II. Plaintiff sues defendant on two unrelated claims, alleging breach of contract for $72,000 and a past due but unrelated bill for $5,000. III. After an auto accident, plaintiff husband sues defendant for $50,000 for loss of consortium, and plaintiff wife sues defendant for her personal injury, for $60,000. All of the above. I and II only. II and III only. II only.

I - there is no subject matter jurisdiction because --the required amount in controversy is not met - the statute requires more than $75,000. II - the amount in controversy is met because --a single plaintiff CAN aggregate claims against a single deft even if the claims are unrelated. III. The required amount in controversy is not met because --separate claims cannot be aggregated to meet the amount in controversy requirement. Therefore, D is the best answer.

Question 4 10 / 10 pts Plaintiff class representatives Pam (from NY) and Paul (from NJ) initiate an action against defendant Insurer (I) from Florida for breach of contract. Frank, another class member who is not a class representative. Paul and Pam each seek $100K from I, but Frank seeks only $70K from I. Does a federal court have subject matter jurisdiction over the matter? No, because Frank is not diverse. No, because Frank's claim does not meet the require amount in controversy. Yes, but only if Frank's claim is dismissed. Yes, the court has subject matter jurisdiction.

The best answer here is D because --only the representatives must satisfy subject matter jurisdiction, and --only one plaintiff must satisfy the amount in controversy under the Exxon case. A is incorrect because --Frank's citizenship is not counted for purposes of diversity. --B is incorrect because --the failure of one claimant to meet the amount in controversy is not relevant as long as --one of the class representatives meets the amount in controversy, which is the case here. C is incorrect because --Frank's citizenship is not counted for purposes of diversity.

Question 4 6 / 6 pts Mary from Virginia brought an action against Lindsey from New York for $500,000 on a breach of contract claim, filed in a New York federal court. Carl from New York intervened as a plaintiff under Rule 24 to assert a claim against Lindsey for $25,000, also for breach of contract. Does the New York federal court have jurisdiction over Carl's claim against Lindsey? Yes, because joinder of Carl's claim would not defeat diversity. Yes, because the parties are diverse. No, because Carl's claim has independent subject matter jurisdiction. No, because Carl's claim would defeat diversity since he is a plaintiff from New York.

The best answer is D because --the final portion of 1367(b) --excludes claims by Rule 24 plaintiffs from enjoying supplemental jurisdiction based on a qualifying diversity claim. Here, Carl is a Rule 24 plaintiff and --thus his jurisdictionally insufficient claim (insufficient amount in controversy and --no diversity) won't qualify for supplemental jurisdiction since the only freestanding qualifying claim is based on diversity jurisdiction. C is incorrect because --Carl's claim does not have independent subject matter jurisdiction. B is incorrect because --the parties are not diverse since --Carl is a plaintiff from New York and Lindsey is a defendant from New York.

Question 2 10 / 10 pts Plaintiffs initiate class action for violation of securities law, seeking damages for fraudulent representations regarding medical device sold by defendant Manufacturer, a publicly traded company. Plaintiffs allege the misrepresentations were made by the manufacturer's Senior VP for Communications in a series of press releases touting M's product. The class representatives are individual and institutional stock purchasers and the plaintiffs' counsel is Milberg Weiss, a well-known securities class action firm. Does the class meet the requirements of 23(a)? Yes, the class meets the requirements of 23(a). No, because the class representatives are not typical. No, because the class representatives are not adequate. No, because the class representatives are not numerous enough.

The best answer is A. --The class is numerous; --joinder is impracticable so D is incorrect. The claims of the representatives are typical as --they are purchasers with claims similar to those of absent class members so B is incorrect. There is --a common question of law and fact which --if fraud, and --adequacy is met by their counsel and --a viable representative, so C is incorrect.

Question 4 4.5 / 4.5 pts Which of the following are considered "final" for purposes of appeal, assuming the matter is not certified for appeal? I. Order denying a motion to dismiss for lack of personal jurisdiction. II. Order granting a motion to transfer venue to another jurisdiction. III. Order dismissing plaintiff's complaint under Rule 37 for failure to comply with discovery. All of the above are final. I and II. II and III. III only.

The best answer is D because (I) is not final; (II) is not final unless certified for appeal; and (III) is final. Accordingly, A, B and C are incorrect.

Score for this quiz: 30 out of 50 Submitted Sep 2 at 9:28am This attempt took 14 minutes. Question 1 0 / 10 pts Plaintiff Pat initiates a class action against the University alleging that U's affirmative action policy is unconstitutional. Pat alleges she was denied admission to U and other minority students with lower scores were admitted. Pat seeks injunctive relief precluding future enforcement of the policy, and damages based on the difference in income Pat will earn from graduating from a lower ranked law school. Assuming the court certifies a class, what type of class is appropriate? A limited fund class and damages class. A damages class and injunction class. A damages class only. An injunctive class only.

The correct answer is C. The court cannot certify injunction class action unless --injunctive relief is the predominate remedy. Here she --also seeks damages, which --probably predominate over the injunction, --therefore B and D are incorrect. This is not a limited fund, so A is wrong.

Question 2 10 / 10 pts Polly is the widow of Jake who died when a truck driven by an employee of ABC, Inc. collided with Jake's car. Before he died, Jake initiated a lawsuit against ABC for personal injuries sustained in the accident. A jury concluded that ABC's driver was not negligent and thus issued a verdict in ABC's favor. After Jake passed away from his injuries, he sustained in the accident, Polly initiated a wrongful-death suit against ABC in Pennsylvania federal court. ABC moves to dismiss the action, asserting that the previous personal injury action involving Jake precludes Polly's wrongful death suit. How should the court rule? Deny the motion, because Polly is proceeding on her own behalf. Deny the motion because Jake did not represent Polly's interests in the earlier case. Grant the motion because Polly litigated the prior action through Jake as a proxy. Grant the motion because Polly is suing as Jake's survivor.

Under the Restatement (Second) of Judgments, § 46, D is the correct answer. A is incorrect because --she is suing as Jake's survivor. B is incorrect because --even though Jake was not representing Polly's interests, --their interests are identical, --with the same rights being asserted in each action. C is incorrect because --there is no indication Polly acted to litigate the case through Jake as a proxy.

Question 21 42 / 45 pts Jack and Jill, residents of Nevada, filed an action in the United States District Court for the Northern District of California, against their former son-in-law, Damien, for wrongful death, alleging that Damien killed their daughter Mary and Mary's unborn child. Damien, a college professor in the Bay Area, alleged that Mary became despondent and committed suicide after Damien disclosed to Mary his affair with a student, Sally. Mary disappeared on New Year's Day, and joggers came upon her body and that of her unborn child two months later, floating in San Francisco Bay. Through his employer, Damien purchased life insurance from Federal Life Insurance Company, a Delaware Corporation, on Mary's life, naming Damien as beneficiary, with a $1 million death benefit. An insurance policy exclusion provided that a beneficiary who murders the insured may not recover the policy proceeds. Shortly after Mary's disappearance, Jack and Jill saw Sally speaking about her affair with Damien on television. Jack and Jill believed that Sally may have killed Mary, so they hired Carly, a well-known lawyer, to investigate. Carly hired a private investigator, Irving, who at Carly's direction, interviewed all of Mary's friends and neighbors, including Alison, Mary's best friend. Alison told Irving that the day after Mary disappeared, Allison received a note from Mary in her mailbox, indicating that Damien's mistress Sally had shown up at Mary's front door with Sally's new baby, Damien, Jr., in tow. The note said that was distraught to the point of wanting to end her own life. Irving summarized the interview with Alison, including her statements about the note, in a written report to Carly. Reading the report, Carly underlined and marked key sections, and noted her thoughts regarding case strategy in the margin. After initiating the wrongful death action, Carly received a document request from Damien's counsel requesting "all documents which evidence, reflect, refer or relate to the possible suicide of Mary." Carly is inclined to object to the request on the grounds that Irving's report is protected from discovery by the work product doctrine. Please advise her of her obligations in responding to the document request, and of the merits of the work product objection.

Your Answer: Carly, I understand that you are requesting my advice on responding to the document request from Damien's (D) counsel. Since the documents are the result of Irving's investigation and interviews in anticipation of litigation, the documents would be subject to protection from the court as being privileged. Release of these documents would compromise the litigation. Even more than that, the fact that you have marked key sections of the notes and entered your thoughts on litigation strategies would warrant increased protection. Under the work-product doctrine, your notes represent your mental impressions, opinions and theories of the case, and would be considered items of absolute privilege and are protected from discovery. I would advise you to respond to the document request by explaining the nature of the documents and objecting to providing these investigation notes and any other documents that would fall into the same category of absolute privilege. Respectfully, John

Question 22 45 / 45 pts The state prosecuted Damien for the murder of Mary and Mary's unborn child in State v. Damien, a criminal proceeding, and the jury found Damien not guilty. Damien then went to trial in Jack & Jill v. Damien, the civil action brought against Damien by Jack and Jill, and the jury found Damien liable for the wrongful death of Mary and the unborn child, and awarded Jack and Jill $1 million in damages. The court entered judgment in favor of Jack and Jill based on the verdict. Federal then filed an action against Damien to recover the $1 million death benefit that Federal paid to Damien, claiming the policy exclusion applied. In Federal v. Damien, Federal filed a motion for summary judgment, asserting preclusion. Damien also filed a motion for summary judgment, also asserting preclusion. Describe the grounds for each motion and discuss the merits of each motion.

Your Answer: Federal v. Damien Federal's motion for summary judgment Summary judgment is available when there is no genuine dispute of material fact. Here, Federal filed a motion for summary judgment, asserting issue preclusion based on the outcome of Jack & Jill v. Damien. The jury in that trial found Damien liable for the wrongful death of Mary and her unborn child, and awarded Jack and Jill $1 million in damages. Federal had paid Damien $1 million in death benefit, and are claiming return of that amount because it was not to be paid if the beneficiary - Damien - was responsible for the murder of Mary. Offensive nonmutual issue preclusion can be applied by a plaintiff to establish liability of the defendant if the issues are related and it would not be unfair to the defendant. Issue preclusion would apply the results of the J&J trial to the Federal trial, since the issues are related and only one party - Damien - would be related to both trials. In this case, the plaintiff is using issue preclusion, which would be an offensive, nonmutual use of issue preclusion. Damien would argue that issue preclusion would be improper due to the fact that he was found not guilty of murder in the criminal trial and it would not be fair to him to use it to avoid a jury trial based on a single adverse verdict in a civil action. The court may find that there is enough conflicting evidence to deny the motion. Damien's motion for summary judgment Damien attempts to use defensive, nonmutual issue preclusion to refute his liability for wrongful death and justify his motion for summary judgment. He bases his motion on the fact that he was found not guilty of murder in the criminal trial. Although the issues are not exactly the same, they are related. Federal would assert that his motion for summary judgment could not be based on the outcome of the criminal trial, as its use is to find no liability for wrongful death based on that jury's verdict. Also, the fact that the J&J trial found that Damien was liable for the wrongful death, which supports Federal's assertion that Damien was, indeed, liable and that the death benefit should be revoked. The court is likely to deny Damien's motion for summary judgment. Quiz Score: 141 out of 180

Question 26 80 / 80 pts Patrick and Danielle were students together at a Southern California law school. After graduation, they both took and passed the California Bar Examination. Danielle then returned to her home state of Texas, where she decided to become an investment counselor in Dallas. Patrick remained in Southern California. Several years later, Patrick contacted Danielle to let her know that he would be visiting Texas for a legal seminar and would like to get together. Patrick went to Dallas, met with Danielle at her office, and learned about her work as an investment counselor. Over dinner in Dallas, Danielle convinced Patrick to invest in WellCo, a Texas corporation. Danielle told Patrick that she could almost guarantee that WellCo stock would double in value, and that several of their other former law school classmates still practicing in Southern California also purchased WellCo stock through Danielle. After Patrick returned home, he purchased 50,000 shares of WellCo stock at $2.00 per share. The stock became worthless, but not before Danielle sold all of her shares at a profit. Patrick learned that Danielle was WellCo's primary shareholder. Patrick filed a fraud suit against Danielle in federal district court in California. Danielle challenged 1) personal jurisdiction, 2) subject matter jurisdiction and 3) venue. How should the court rule? Analyze the issues and explain your answer.

Your Answer: Patrick (P) v. Danielle (D) Is there PJ over D? Is there SMJ over D? What is proper Venue? Personal Jurisdiction Traditional bases - PJ is available where --D consents, --is present in the forum state and --served there, or --is resident of forum state Here, there is no indication that D consented to the lawsuit. Furthermore, D is not domiciled in CA, because she is a resident of TX. It is presumed that D was served in TX. Since the traditional bases for PJ have not been met, we will consider minimum contacts. CA has a long arm statute that extends to the limit of due process. Therefore, the claim is valid for a D in TX. Minimum contacts can establish PJ where --D has continuous and systematic contacts in the forum state, --the claim relates to those contacts, --D purposely avails herself to the benefits and protections of the forum state, --it is foreseeable that D could be haled into court in the forum state, and --the action is fair and reasonable, and --comports with due process. Here, the facts indicate that D has significant contacts with former classmates in CA and has sold much stock to those contacts. Those contacts relate to P's claim because the sale of the worthless stock that was sold to her classmates and P. Furthermore, D has purposely availed herself to the benefits and protections of the forum state by benefitting from the revenue she earned from the stock sales as the principal stockholder of the company. It could be anticipated due to her tortious activity in the forum state that she could be haled into court there. The litigation would be fair and reasonable because the interests of CA would be concerned with justice for her customers in CA, and her travel to CA would not be a burden for court appearances - all other possible participants likely would be in CA. Therefore, all the elements of PJ have been met and the court should dismiss D's challenge. Subject matter JDX SMJ is available in a diversity claim where the P and D are residents of different states and the amount in controversy exceeds $75,000. Here, the facts state that P stayed in CA, and it is presumed that he is a resident there because no other states were indicated. The facts also state the D moved back to TX, because that is her home state. P remained in CA, and the facts state that he "returned home" to CA. Thus P is a resident of CA. P filed the claim because he lost 50,000 shares of stock that he bought at $2 per share, Thus, his loss due to fraud would amount to $100,000, which would exceed the $75,000 required for diversity jdx. Therefore, the case meets the complete diversity requirements for SMJ. Therefore the court should dismiss D's challenge. Venue A proper venue exists where --D is resident of the federal district of the state, or --the location where the substantial part of the activity claimed occurred, or --any district where PJ is valid. Here, D is a resident of TX, and her fraudulent activity occurred there, because she handled all the stock sales in TX and collected the revenue there as the primary stockholder. On the other hand, P could argue that he purchased the worthless stock in CA, and his losses were incurred there, and that CA also would be a proper venue for the case. Thus, both TX and CA could be considered proper venues. Therefore, the CA court could rule that the case be litigated in the federal district court in CA. Quiz Score: 148 out of 180

Question 7 4 / 4 pts Plaintiff Polly, from California, has filed suit in federal district against her former attorney Arthur, also a California resident, for legal malpractice, claiming that Arthur, who represented Polly in a patent infringement matter, committed legal malpractice (essentially professional negligence) based on his failure to assert a particular argument in connection with her prior patent infringement claim. Arthur moved to dismiss the claim, due to lack of subject matter jurisdiction. Which of the following should the court consider? I. Whether the federal issue is substantial. II. Whether the patent law issue is actually disputed. All of the above. I only. II only. None of the above.

A is the best answer --under the Supreme Court's decision in --Grable v. Darue which states that --both factors will be considered. Accordingly, B, C and D are incorrect. Grable Standard - Key question in determining whether state-law claims involving federal issues qualify for federal question jurisdiction is "does a state-law claim necessarily raise a stated federal issue, actually disputed and substantial, which a federal forum may entertain without disturbing any congressionally approved balance of federal and state judicial responsibilities?"

Question 12 0 / 4 pts Plaintiff Pierce sued defendant Della for defamation based on some derogatory and false comments Della published in her blog about Pierce's business practices. Pierce filed the case in federal court because the parties were diverse. Della alleged that the state's one year statute of limitations applied to her claim and that Pierce had filed the claim 14 months after she made the alleged statements. Pierce claimed that the federal court was not bound by the state statute of limitations and could take the equities of the situation into consideration. Della filed a motion to dismiss the case based on the one year statute of limitations. How should the court rule? The state statute of limitations applies because it is outcome determinative. The state statute of limitations applies because it is procedural. The state statute of limitations applies because it is substantive. The state statute of limitations need not be applied to preserve federal uniformity.

A is the best answer based on --Guaranty v. York, which held that the state statute of limitations is outcome determinative, and --bound up with the rights of the parties , while --the federal interest is relatively weak and --federal uniformity is not required. B is incorrect because --York said --it is immaterial whether the state statute or rule is --labeled as substantive or procedural, since the point is --whether it is outcome determinative, --which is the case for a statute of limitations. C is incorrect for the same reason. D is incorrect because --there is no need for federal uniformity and --the federal interest is relatively weak.

Question 20 0 / 4.5 pts Wife #2 intervened in Wife #1's suit against Husband's life insurance company, claiming husband changed beneficiaries before his death to Wife #2. Wife #2 moved for summary judgment, submitting the affidavits of Bart, the employee who processed the change of beneficiary form, and Attorney, who indicated that Husband changed his will to benefit Wife #2 around the same time, and sent Attorney a letter saying he wanted to update both his will and his life insurance to benefit Wife #2. Wife #1 opposed the motion alleging Bart and the Attorney were biased. The court should: Grant the motion, without specific evidence of bias. Grant the motion, because the Attorney is a reliable witness. Deny the motion, because there is an issue of credibility. Deny the motion, because Wife #1 has raised a triable issue of fact.

A is the best answer based on Lundeen v. Cordner, where the court held on these facts that Wife #1 had failed to provide specific facts alleging bias and therefore the motion was granted. B is incorrect because an attorney may or may not be a reliable witness. C is incorrect because Wife #1 has not submitted sufficient evidence to raise an issue of credibility. D is incorrect for the same reason, and because Wife #1 has not submitted any evidence that would contradict the evidence submitted by Wife #2.

Question 21 4 / 4 pts Able, from the Northern District of New York, sues Baker and Cosmo in federal district court. Cosmo is from the Southern District of New York. Baker is from the Eastern District of New York. Where is venue proper? Either the Southern or Eastern District of New York Only the Northern District of New York. In the Northern, Southern or Eastern District of New York. Only in the Southern District of New York.

A is the best answer because --all of the defendants --reside in the same state, so venue is proper --in any of the districts in which any of the defendants reside. --Thus, venue would be proper either in the --Southern or Eastern Districts of New York. --Venue is not proper --where the plaintiff resides, so --the Northern District is not a proper venue, --making B and C incorrect. D is incorrect because --venue would also be proper in the district where Baker resides, --the Eastern District of New York.

Question 23 0 / 4 pts Plaintiff Patrice, a resident of Vermont, filed a lawsuit against defendant Donald, a resident of New Hampshire, over Donald's remodel of Patrice's home. Patrice filed the case in New Hampshire in federal court. She alleged that Donald's shoddy will cost at least $70,000 to repair. She also seeks recovery of her attorneys' fees, and alleges she gave her attorney a $10,000 retainer. Her complaint also seeks recovery of costs and interest. Donald moved to remand the case to state court, alleging a lack of jurisdiction. How should the court rule? Grant the motion because the court has no subject matter jurisdiction. Grant the motion because the court has no personal jurisdiction. Deny the motion because the parties are diverse. Deny the motion because the allegations satisfy the amount in controversy.

A is the best answer because --although the parties are diverse, --the amount in controversy is not met, --since the court cannot consider --interest, --attorneys' fees or --costs in evaluation of the amount in controversy, so --her allegation of at least $70,000 does not meet the requirement that the amount in controversy exceed $75,000. B is incorrect because --the court does not address personal jurisdiction --on a motion to remand. C is incorrect because --although the parties are diverse, --the amount in controversy is not met, so --the motion should not be denied. D is incorrect because --the amounts alleged do not meet the amount in controversy requirement, since --the attorneys' fees, costs and interest cannot be included.

Score for this attempt: 148 out of 180 Submitted Aug 15 at 10:31pm This attempt took 120 minutes. Question 1 4 / 4 pts Plaintiff Paul, a resident of California is injured when the bus he is riding in is involved in an accident. Paul sues Driver and Bus Co., the corporation that owns the bus, for negligence, filing the case in federal court in the state of California. Bus Co. is incorporated in Delaware, and has its principal place of business and corporate headquarters in the State of California. The accident occurred in the State of California. Driver is also a resident of California. Which of the following are correct? The court has general jurisdiction over Bus Co. and Driver. The court has subject matter jurisdiction based on diversity. The court has only specific jurisdiction over Driver and Bus Co. The court has subject matter jurisdiction based on a federal question.

A is the best answer because --both defendants are present in California because --Driver is domiciled there and --Bus Co has its principal place of business there. B is incorrect because --the parties are all from California. C is incorrect because --the court has general jurisdiction based on domicile. D is incorrect because --the case involves negligence which is a --common law doctrine and --not based on a federal statute.

Question 2 6 / 6 pts Plaintiff Pat, a citizen of Florida, sued defendant Dan, a citizen of New Mexico. The court will have subject matter jurisdiction over the case if: I. The amount in controversy exceeds $75,000. II. Pam's claim is based on a federal statute and the amount in controversy is $50,000. Both I and II. I only. II only. None of the above.

A is the best answer because --the court will have subject matter jurisdiction based on diversity under scenario (I) and --the court will have jurisdiction based on a federal question, where not amount in controversy is required. Therefore B, C and D are incorrect.

Score for this quiz: 40 out of 50 Submitted Sep 26 at 3:10pm This attempt took 13 minutes. Question 1 10 / 10 pts Defendant tire manufacturer is sued by Plaintiff, a mechanic who suffered a head injury in a shop when the tire exploded as a result of a defect. The evidence from the Manager at the scene was that the tire was a whitewall. Plaintiff testified the tire was a blackwall. Defendant only manufactures whitewall tires. After this evidence, Defendant moves for a directed verdict. How should the court rule? Deny the motion because there is a conflict in the evidence. Deny the motion because Plaintiff is more credible than the Manager Grant the motion based on plaintiff's testimony. Grant the motion because no reasonable jury could believe the Manager.

A is the best answer because --the standard is --whether a reasonable party --would have a sufficient evidentiary basis to --find for the party on the issue and --here, the jury could believe the manager. B is incorrect because --the test is not who is more credible. C is incorrect because --the jury could reasonably believe the manager, --despite the plaintiff's testimony. D is incorrect because --the jury could reasonably believe the manager.

Question 19 0 / 4 pts Plaintiff Preston, a resident of Idaho and professional photographer, sued defendant Darcy, a resident of New York, and defendant Darwin, a resident of Florida, over several incidents that occurred at an amusement park when the three vacationed at a resort in California. Preston claimed that Darcy stole Preston's video camera and related equipment from his hotel room, worth $50,000, and that Darwin committed the tort of conversion after he took an expensive ring worth $30,000 from Preston after he passed out in the hotel bar. Preston filed the case in federal court in Florida. Darcy moved to dismiss for lack of personal jurisdiction. Darwin moved to dismiss based on lack of subject matter jurisdiction. How should the court rule? Grant both motions. Grant Darcy's motion, but deny Darwin's motion. Grant Darwin's motion, but deny Darcy's motion. Deny both motions.

A is the best answer because --there is no personal jurisdiction in Florida over Darcy, --given the incidents occurred in California and --Darcy lives in New York; and, --there is no subject matter jurisdiction over Darwin since the claim against him is based on --conversion of the ring, because --although the parties are diverse, --the claim is under $75,000 and --plaintiff cannot aggregate --the claim against Darcy to meet the amount in controversy.

Question 3 4.5 / 4.5 pts Art sued Dan in federal court for patent infringement involving several features of a popular phone widely used at the time. At trial, the jury found in favor of Dan. After the verdict Art spoke with several jurors who indicated that during jury deliberations, several jurors indicated that they had looked at their own phones to determine certain aspects of the alleged patent infringement claims. This violated specific instructions from the judge. The trial court can best address this through what procedural vehicle? Motion for new trial. Motion for judgment notwithstanding the verdict. Only through an appeal. All three of the above procedural devices.

A is the best answer because jury misconduct is a basis for a new trial. B is incorrect because the standard on a JNOV motion is that there is no reasonable basis for the jury's decision, which does not address juror misconduct. C is incorrect because although juror misconduct can be addressed on appeal, it can also be addressed at the trial level (which is the call of the question here) through a motion for a new trial. D is incorrect for the reasons stated.

Question 2 0 / 4.5 pts Alan brought a claim for intentional infliction of emotional distress based on a statement made by his supervisor Sam during a performance review indicating that Alan had not properly managed his time during his last period of review. The two worked remotely and were located in different states. Alan sought $100,000 in damages. Alan did not suffer any medical symptoms and did not seek any medical attention due to stress, and continued working during the period following the review up through the filing of the lawsuit. A claim for intentional infliction of emotional distress requires extreme and outrageous conduct causing severe emotional distress. At trial, after Alan testified to Sam's conduct, Sam moved for a directed verdict/ judgment as a matter of law. How should the court rule? Grant the motion unless reasonable jurors could differ as to the result. Grant the motion because a verdict for Alan would shock the conscience. Deny the motion because there is a genuine issue of material fact. Deny the motion because Alan has a right to a jury trial based on the U.S. Constitution.

A is the best answer because the standard for a directed verdict is that --no reasonable juror could find for a party, which is the case here. B is incorrect because --the test for a directed verdict is whether a reasonable juror could find for the party, not whether a verdict would shock the conscience, which is one of the tests for a new trial. C is incorrect because --there is no genuine issue on these facts, and --the test stated is used on a summary judgment motion, not a motion for a directed verdict. D is incorrect because --a directed verdict motion allows the court to take the case from the jury where reasonable jurors could not differ.

Question 5 6 / 6 pts A female plaintiff from New York sues her employer, a French company, filing suit in New York for sex discrimination that occurred during plaintiff's visits to corporate headquarters in France. French law requires a higher burden of proof for such claims. How should the court rule on a motion to dismiss the case for forum non conveniens? Grant the motion because there is a remedy available under French law and most of the events occurred in France. Grant the motion because French law is more favorable to the defendant. Deny the motion because there is effectively no remedy available in France. Deny the motion because the plaintiff's choice of forum is determinative

A is the best answer given that French law provides some remedy, albeit with a higher burden of proof, and --that most of the evidence and witnesses and --premises where the events occurred are in France. B is incorrect because --a more favorable law alone is not a basis for moving the case to another jurisdiction. --C is incorrect because --there is an available remedy in France, though --with a higher burden of proof, so --the plaintiff is able to vindicate her rights conceptually. D is incorrect because --the plaintiff's choice of forum is not a determinative factor, although --convenience of the parties is an important consideration and --the plaintiff's choice of forum is a factor Quiz Score: 24 out of 30

Question 3 0 / 6 pts In State A, there lives a former employee of a corporation who once lived and worked for the company in its State B headquarters, where the corporation is incorporated. The former employee sued the corporation in State A federal court, asserting a claim for wrongful termination. To achieve service of process, the former employee served the driver of one of the corporation's delivery trucks while it was stopped at a gas station in State A. Is the exercise of personal jurisdiction over the corporation in State A be constitutional? No, because corporation did not have minimum contacts with State A related to the claim. No, unless the corporation consented to personal jurisdiction in State A. Yes, because the driver was served in State A. Yes, because the employee lives in State A.

A is the best answer here because --International Shoe requires minimum contacts, and --for specific jurisdiction, --those contacts must be related to the claim. B is incorrect because --jurisdiction may still be constitutional --even if the defendant did not consent to it, if --the defendant had minimum contacts --related to the claim. C is incorrect because --there is no indication that --the driver was a valid agent for service of process. D is incorrect because --personal jurisdiction over the corporation --does not depend on where the employee lives.

Score for this quiz: 24 out of 30 Submitted Aug 1 at 9:34pm This attempt took 12 minutes. Question 1 6 / 6 pts Plaintiff from California sues defendant from Arizona and defendant from Nevada, in Arizona state court seeking $200K on a federal civil rights claim. Both defendants timely seek removal. Can the case be removed? Yes the case can be removed because the court has federal question jurisdiction. Yes the case can be removed based on diversity jurisdiction. bars removal in diversity cases No, the case cannot be removed because one defendant resides in Arizona where the case is filed. No, the case cannot be removed due to the well pleaded complaint rule.

A is the best answer here because --the case can be removed since --both defendants are in agreement and the in state removal rule is not a barrier to removal based on a federal question. B is incorrect because, --although the parties are diverse and required amount in controversy is met, --there is an in-state deft so --the case cannot be removed based on diversity. C is incorrect because the in state defendant rule --bars removal in diversity cases, but --not in federal question cases. D is incorrect because --looking at the well-pleaded complaint, --there is a federal question which is --a valid basis for removal here.

Question 17 0 / 4 pts Plaintiff Pearl, from California, sued defendant Dante, a resident of Nevada, for negligence as a result of an automobile accident that occurred in Nevada, filing the case in Nevada state court. Pearl alleged $100,000 in damages for personal injury and harm to her vehicle. Dante removed the case to federal court. A strong federal policy requires a unanimous jury verdict in a diversity case, although there is no federal rule on point. The State of Nevada has a rule requiring a less than unanimous jury verdict. At trial, must the jury's verdict be unanimous, or should the federal court follow the state rule? The verdict must be unanimous, due to the strong federal policy. The verdict must be unanimous, because the issue is heavily outcome determinative. The state rule should apply, because there is no federal rule on point. The state rule should apply, because the issue is heavily outcome determinative.

A is the best answer here under the Byrd and York cases, --given the strong federal policy and --the fact that the choice would not be outcome determinative since --it is not likely to influence who will prevail in the suit. B is incorrect because --the choice is not outcome determinative. C is incorrect because --the strong federal policy cannot be disregarded under Byrd --even though there is no federal rule on point. --D is incorrect because --the issue is not outcome determinative.

Question 20 0 / 4 pts Plaintiff Patton worked for defendant Power Company. Patton filed a lawsuit in federal court claiming he was an employee while Power Company claimed that Patton was an independent contractor. Under a strong federal policy of having factual matters decided by a jury, the issue of whether Patton was an employee or independent contractor would be decided by the jury. Under state law, the issue would be decided by the judge. There is no federal rule on point. Who should decide the issue, judge or jury? The jury should decide because it is a matter of strong federal policy. The judge should decide based on the outcome determinative test. The jury should decide because there is no federal rule on point. The jury should decide based on the issue is outcome determinative.

A is the best answer under the Byrd v. Blue Ridge decision. B is incorrect because --the choice is not very outcome determinative. C is incorrect because --the strong federal policy weighs heavily in favor of the jury deciding. D is incorrect because --the choice is not very outcome determinative.

Question 19 4.5 / 4.5 pts Leon sued his employer Equi-Fax in federal court for age discrimination based on federal law and intentional infliction of emotional distress based on several age-related comments following which Leon was terminated in connection with a reduction in force in which 20% of the company's employees were let go. At trial, Equi-Fax made a motion for directed verdict, which the court denied. Leon prevailed and the jury awarded Leon $500,000 in compensatory damages and $1 million in punitive damages. What procedures are available to Equi-Fax to address the jury's decision, including the merits of the case and the excessiveness of the verdict? I. Motion for judgment notwithstanding the verdict. II. Motion for new trial. III. Appeal. All of the above. Only I and II. Only II and III. Only I and III.

A is the best answer, as all of these procedural devices could be used given that a JNOV would allow the court to enter a verdict in favor of Equi-Fax, and could attack the damages in a motion for new trial as well as on appeal. Therefore B, C and D are incorrect.

Score for this quiz: 30 out of 50 Submitted Sep 25 at 10:39pm This attempt took 12 minutes. Question 1 10 / 10 pts Paul sues Devin for damages arising out of a vehicle collision. In defense, Devin pleads that the suit has been compromised by a settlement agreement. In his answer, Paul denies this claim. Devin moves for summary judgment and attaches a copy of the settlement agreement and an affidavit by Devin authenticating the signatures. Paul does not respond. The court should: Grant the summary judgment motion because there is no genuine issue of material fact. Grant the summary judgment motion because the court must view the evidence in light most favorable to the moving party. Deny the summary judgment motion because there is an issue of credibility. Deny the summary judgment motion to give Paul another chance to respond.

A is the best answer, because --Devin met the burden of proof as to the asserted defense. B is incorrect because --the court must view the evidence in the light most favorable --to the non-moving party. C is incorrect because --no issue of credibility is raised by these facts. --D is incorrect because --there is no basis for delaying the hearing on the motion.

Question 2 10 / 10 pts Pam, a California resident, was injured by a defective can opener that sliced her finger causing permanent damages. She filed a product liability claim against Retailer, a Delaware with its principal place of business in Delaware, for $100,000. Retailer sued Manufacturer, a New York corporation with its principal place of business in New York, for indemnity. Pam then brought a product liability claim against Manufacturer for $100,000. Manufacturer asserted a claim against Retailer for negligence arising out of Retailer's handling of the product, seeking $100,000. Which of the following is accurate? All claims are permissible pursuant to applicable joinder rules, and the court has subject matter jurisdiction. Retailer's claim against Manufacturer is properly joined, but the court has no subject matter jurisdiction for the claim. Pam's claim against Manufacturer was not properly joined. Manufacturer's claim against Retailer is not properly joined.

A is the best answer. Pam's claim for product liability vs. Retailer qualifies for --subject matter jurisdiction based on diversity. Retailer's claim against Manufacturer is --a proper third party impleader claim, and --has subject matter jurisdiction based on --diversity, therefore B is incorrect. Pam's claim against Manufacturer is --properly joined under Rule 14 and --has subject matter jurisdiction based on --diversity therefore C is incorrect. Manufacturer's claim against Retailer is --proper under Rule 13 and --has subject matter jurisdiction based on --diversity therefore --D is incorrect.

Question 4 0 / 6 pts Jon Peters, a senator from Nevada, brought a defamation lawsuit against the Los Angeles Times, a California corporation, in California superior court, arising out of an article published in the Times involving aspects of the senator's private life. In its Answer, the Times asserted that its publication was protected by the First Amendment. The Times removed the case to federal court in Nevada and Peters moved to remand. How should the court rule? Grant the motion because the Times is a citizen of the forum state. Grant the motion because the case should have been removed to Nevada. Deny the motion because the case arises under federal constitutional law. Deny the motion because the court has jurisdiction based on diversity.

A is the correct answer because --an in-state defendant --cannot remove a case to federal court. B is incorrect because --any removal is to the court in the same state, --not in another state. C is incorrect because --the constitutional issue is in the answer which --does not satisfy the well pleaded complaint rule. D is incorrect because, --although there is diversity jurisdiction, the case cannot be removed due to --the rule barring removal by in state defendants.

Question 5 4 / 4 pts Plaintiff Portia, a resident of California, sued defendant Donald, a resident of Nevada, and his company, DonCo, which was incorporated in Nevada, and had its principal place of business in California. Portia filed the case in federal court, and alleged two claims of copyright infringement against Donald, each for $40,000, and a single claim of copyright infringement for $80,000 against DonCo. Which of the following are true? The court has subject matter jurisdiction over all claims based on diversity. The court has subject matter jurisdiction over all claims based on federal question. The court has subject matter jurisdiction over the claims against DonCo but not over the claims against Donald. The court has no subject matter jurisdiction.

B is the best answer because --all of Portia's claims involve copyright infringement, --which is a federal question. A is incorrect because --given that DonCo has its principal place of business in California, --where Portia resides, there is --no diversity between all plaintiffs and all defendants. C is incorrect because --the court does in fact have --subject matter jurisdiction over the claims against Donald --based on a federal question. D is incorrect because --the court has subject matter jurisdiction over all claims based on subject matter jurisdiction. [FED Q]

Question 4 10 / 10 pts Howard sued Joe for breach of contract, seeking specific performance (equitable relief) and money damages. Is Howard entitled to a jury trial? Which issues must the jury resolve? A jury must resolve all of the issues raised by Howard. A jury must resolve the issue of damages and whether there was a breach of the contract. A jury must resolve whether there was a breach of the contract and the court must address damages and specific performance. A jury must resolve whether there was a breach of the contract and specific performance, but the court can award damages.

B is the best answer applying Beacon Theaters to the facts, because --the parties have a right to --have the jury resolve the issue of --whether there was a breach of contract and --whether Howard is entitled to damages (damages are legal relief). Once the jury determines that --a breach of contract has occurred, --the judge may then --determine whether --Howard is entitled to --a decree ordering specific performance Therefore A, C and D are all incorrect.

Question 3 10 / 10 pts Art sued Dell for breach of contract. Dell brought a claim against Tom for indemnity, arising out the breach of contract claim brought by Art against Dell. Dell also brought a claim against Art, claiming Art had breached the same contract, and seeking damages. Which of the following is correct? Dell's claim against Tom is based on impleader, and Dell's claim against Art is a cross-claim. Dell's claim against Tom is based on impleader, and Dell's claim against Art is a counterclaim. Dell's claim against Tom is a cross-claim, and Dell's claim against Art is a counterclaim. Dell's claim against Tom is a cross-claim, and Dell's claim against Art is a cross-claim.

B is the best answer as --Dell's claim against Tom is --bringing Tom into the action as a third party, which is --impleader, and --Dell's claim against Art is a counterclaim --against an opposing party, --not a cross-claim against a co-party. Accordingly, A, C and D are incorrect.

Score for this attempt: 141 out of 180 Submitted Sep 26 at 11:06pm This attempt took 118 minutes. Question 1 4.5 / 4.5 pts Robert sued his former business partner Dillon for breach of their partnership agreement and dissolution of the partnership, as well as conversion, based on allegations of embezzlement by Dillon. Robert sought discovery in the lawsuit including financial records from Dillon's bank during the time period of the alleged embezzlement. The Bank was not a party to the case. Based on partnership records, Robert believes that the Bank records will show that Dillon had been siphoning money from the partnership account into Dillon's personal account. Which of the following is correct? Robert may obtain records from the Bank through a document request, and the records are relevant to the subject matter of the action. Robert may obtain records from the Bank through a subpoena, and the records are relevant to his claims. Robert may not obtain records from the Bank because the records are not relevant to his claims. Robert may only obtain records from the Bank by filing a motion with the court seeking the information.

B is the best answer as it appears the records are relevant to the claim of conversion and embezzlement, and the Bank is not a party, so a subpoena is needed to obtain the records. A is incorrect because --a document request can be issued only to a party, and --the relevance is determined based on the claims, not the subject matter of the action generally. C is incorrect because --the records are relevant. D is incorrect because --a motion is unnecessary.

Question 16 0 / 4.5 pts Frank, George and Hal have hired you to advise them whether they can bring a class action against their former employer for discrimination. Frank wants to allege discrimination based on race. George wants to allege age discrimination. Hal wants to allege discrimination based on his sexual orientation. At this time they are not aware of others who have encountered similar discrimination by their employer. What advice do you have for them? I. Their claims do not share common questions of law. II. They cannot meet the requirements for numerousity. III. Interpleader would be a better procedural device. All of the above. I and II only. II and III only. I and III only.

B is the best answer as their claims of distinct types of discrimination will not meet the standard of common questions of law, and only three plaintiffs would not meet the requirement of numerousity; however interpleader is not appropriate here because there is no stake and no stakeholders with claims to the stake. Accordingly, A, C and D are incorrect.

Question 18 4 / 4 pts Plaintiff Penelope and her daughter Plaintiff Pippa, both residents of Oregon, brought suit seeking damages based on negligence under state law against defendant Devlin, a resident of Washington State, and restaurant owner, who served Penelope and Pippa tainted tuna fish when they ate at the restaurant, causing severe illness to Penelope. Penelope was hospitalized and suffered damages based on her medical bills of over $100,000. Thankfully Pippa only suffered a rash, and her medical treatment cost under $3,000. Does the court have subject matter jurisdiction over the case? The court has original subject matter jurisdiction over both claims based on diversity. The court has original subject matter jurisdiction over Penelope's claim, and supplemental jurisdiction over Pippa's claim. The court has original subject matter jurisdiction over Penelope's claim, but no subject matter jurisdiction over Pippa's claim. The court has no subject matter jurisdiction over either claim.

B is the best answer based on Exxon v. Allapattah, which held that --as long as one claim has subject matter jurisdiction, --there is supplemental jurisdiction over the other plaintiff's claims --even if the amount in controversy is not satisfied. A is incorrect because --the court does not have original jurisdiction over the Pippa's claim due to --the amount in controversy being under $75,000. C is incorrect because --the court has supplemental jurisdiction over Pippa's claim. D is incorrect because --the court has original subject matter jurisdiction over Penelope's claim, and --supplemental jurisdiction over Pippa's claim.

Score for this quiz: 24 out of 30 Submitted Jul 18 at 9:37pm This attempt took 12 minutes. Question 1 0 / 6 pts Over which of the following defendants does the court most likely have personal jurisdiction, assuming those defendants have no other contacts with the forum state? I. A defendant served with process during a layover in the forum state. II. A person served with process after they mistakenly crossed into the forum state while lost. Both I and II. I only. II only. None of the above.

B is the best answer because --a court will likely have personal jurisdiction over a person who is --voluntarily and knowingly present in the state, --which likely must be intentional, and --is less likely to have personal jurisdiction over a person mistakenly in the state. So A and C are incorrect. D is incorrect because I is correct.

Question 3 0 / 10 pts Bart sued Jan for breach of contract in federal court based on diversity. The parties went to trial in the case and Bart prevailed, however, Jan discovered the jury looked up several aspects of the case on the internet in order to reach the verdict. Jan also believes that the jury's damages award is excessive. Prior to appealing, how should Jan proceed? File a motion for judgment notwithstanding the verdict, and seek a new trial. File a motion for new trial and seek remittitur. File a motion for judgment notwithstanding the verdict, and seek remittitur. File a motion for new trial, a motion for judgment notwithstanding the verdict, and seek remittitur.

B is the best answer because --a motion for new trial can be based on --excessive damages, as well as --jury misconduct, and because --there is no indication Jan filed a motion for directed verdict, so --a motion for judgment notwithstanding the verdict --will not be available. A is incorrect because --a motion for judgment notwithstanding the verdict Is not available given that --there is no indication Jan filed a motion for directed verdict. C is incorrect for the same reason. D is incorrect for the same reason.

Question 5 0 / 10 pts Al, an applicant denied admission to University sued University on grounds of discrimination based on University's admissions policies. Bart, another applicant denied admission, seeks to intervene in the case. There is no federal statute permitting intervention. Bart can intervene as of right: I. If Al does not adequately represent Bart's interests. II. If Bart's rights would be impaired should he not be permitted to intervene. III. If Bart intends to seek the same relief as Al All of the above. I and II only. I and III only. II and III only.

B is the best answer because --intervention as of right requires --an interest in the subject matter of the action, which --Bart has, --inadequate representation of those interests by the existing parties and --potential impairment of the interest, but --does not require that similar relief be sought. Accordingly, A, C and D are incorrect. Quiz Score: 30 out of 50

Question 15 4 / 4 pts Auto Corp., a Delaware Corporation headquartered in Tennessee, purchases an electronic component for the cars it manufactures from Chip Co. located in Japan. The cars are sold all over the United States. After a malfunction in acceleration, Plaintiff Paige sues Auto Corp. and Chip Co. in California, where Paige purchased her Auto Corp. car from an Auto Corp. dealer. The evidence shows that Chip Co. does no advertising in the US, and ships roughly 2% of its product to the US. Which of the following describes the court's personal jurisdiction over the defendants? The court has personal jurisdiction over both Auto Corp. and Chip. Co. The court has personal jurisdiction over Auto Corp. but not Chip Co. The court has personal jurisdiction over Chip Co. but not Auto Corp. The court has no personal jurisdiction over the defendants.

B is the best answer because --it would be unfair to require Chip Co. to defend itself in the US on these facts, which are similar to the Asahi case. A is incorrect because --there is no jurisdiction over Chip Co. on these facts. C is incorrect because --there is jurisdiction over Auto Corp., since --it is present in California, and --has a dealer there, and --sold the car there, but --there is no jurisdiction over Chip Co, for the reasons described. D is incorrect because --there is jurisdiction in California over Auto Corp.

Question 2 10 / 10 pts Bella, a resident of California, filed a claim in federal district court in California alleging breach of contract against Don, a resident of Texas. Don filed a motion to dismiss the case based on failure to state a claim on which relief could be granted. The court denied the motion. Don then filed a motion to dismiss the case for lack of personal jurisdiction. How should the court rule? Deny the motion only if Don had minimum contacts with California. Deny the motion as untimely. Grant the motion if Don did not have minimum contacts with California. Grant the motion if Don was not a resident of California at the time of the contract.

B is the best answer because --the motion is untimely given that Don filed the first motion to dismiss --without raising the personal jurisdiction issue, --thereby waiving the personal jurisdiction issue. A is incorrect, because --the motion should be denied regardless of whether Don had minimum contacts with California. C and D are incorrect because --the motion would be denied for the reasons indicated.

Score for this quiz: 40 out of 50 Submitted Sep 20 at 10:29am This attempt took 13 minutes. Question 1 0 / 10 pts Following an automobile accident between Pat and Debbie, Zurich Insurance questioned Walt, a witness, concerning the accident. Pat sued Debbie and Debbie issued a subpoena to Zurich seeking production of Walt's statement concerning the accident. Zurich had sent the statement to Pat's attorney. Walt is available and could be interviewed by Debbie's attorney. Which of the following is correct? The statement is protected by the attorney client privilege. The statement is protected by the work product doctrine. The statement is protected by both the attorney client privilege and the work product doctrine. The statement is not protected by either the attorney client privilege or the work product doctrine.

B is the best answer because --the statement was prepared in anticipation of litigation, and --there is no undue hardship to obtain the information --through Debbie --having her attorney take a statement from Walt. A is incorrect because --the statement is not a confidential communication --between client and counsel. C is incorrect for the same reason. D is incorrect because --the statement is protected by the work product doctrine for the reasons indicated.

Question 6 4.5 / 4.5 pts Patty, who is from Arizona, filed a lawsuit against Diane, who is from California, for breach of contract in federal district court in California seeking $150,000. The two had done business for years. Patty creates a wide variety of pottery, which Diane sells in several retail outlets in Southern California. Diane filed a counterclaim against Patty for breach of a separate contract involving several paintings, seeking $10,000. Patty moved to dismiss the counterclaim. How should the court rule? Grant the motion because the claim is compulsory counterclaim, but the court has no subject matter jurisdiction. Grant the motion because the claim is a permissive counterclaim without subject matter jurisdiction. Deny the motion because the claim is a compulsory counterclaim, and the court has original jurisdiction. Deny the motion because the claim is a permissive counterclaim and the court has supplemental jurisdiction.

B is the best answer because the claim is a permissive counterclaim, since it is on an unrelated contract, and the court has no supplemental jurisdiction over such claims, and this claim does not qualify for original jurisdiction due to the amount in controversy. A is incorrect because this is not a compulsory counterclaim, since it involves a different contract. C is incorrect because this is not a compulsory counterclaim, since it involves a different contract and because the court does not have jurisdiction. D is incorrect because the court does not have supplemental jurisdiction because the claim relates to a different contract.

Question 8 0 / 4.5 pts Paula, a citizen of California, was vacationing in Hawaii, where she had dinner at a small restaurant owned operated by Art, a citizen of Hawaii. Paula became very ill, requiring hospitalization, and incurring $100,000 in medical expenses. The problem was traced to some vegetables Art had purchased from Van, a California resident, a wholesaler, who purchased the vegetables used to make the dinner from Frank, a resident of California. Paula sued Art. Art impleaded Van, who impleaded Frank. Paula then filed a claim in the same case against Frank. Frank moved to dismiss the claim by Paula and the claim by Van. How should the court rule? Grant both motions. Grant the motion to dismiss as to Paula's claim, but deny the motion as to Van's claim. Grant the motion to dismiss as to Van's claim, but deny the motion as to Paula's claim. Deny both motions.

B is the best answer because there is no supplemental jurisdiction over a non-diverse claim against a third party by a plaintiff, and no original jurisdiction because Paula and Frank are not diverse, but the court does have supplemental jurisdiction over the third party claim brought by Van against Frank. A is incorrect because there is no supplemental jurisdiction over a non-diverse claim against a third party by a plaintiff. C is incorrect because there is no supplemental jurisdiction over a non-diverse claim against a third party by a plaintiff, and no original jurisdiction because Paula and Frank are not diverse, but the court does have supplemental jurisdiction over the third party claim brought by Van against Frank. D is incorrect because the court does have supplemental jurisdiction over the third party claim brought by Van against Frank despite the fact they are not diverse.

Question 8 0 / 4 pts Plaintiff Patrick, a resident of New York, sued defendant Derrick, also a resident of the state of New York asserting two claims: (1) trademark infringement and (2) unfair competition based on a New York statute. Both claims allege that Derrick used a confusingly similar trademark on his product to a trademark used by Patrick. Which of the following are true regarding subject matter jurisdiction? The court has no subject matter jurisdiction. The court has subject matter jurisdiction over (1) based on a federal question and (2) based on supplemental jurisdiction. The court has subject matter jurisdiction only over (1), and not over (2). The court has subject matter jurisdiction over both (1) and (2) based on a federal question.

B is the best answer here because --the court has subject matter jurisdiction over --(1) based on the trademark claim, which is --a federal question and --(2) based on supplemental jurisdiction, because --the claim involves the same case or controversy. --A is incorrect because --the court will have subject matter jurisdiction over both claims. --C is incorrect because --the court has supplemental jurisdiction over (2) since it is --part of the same case or controversy as (1). D is incorrect because --(2) involves a state law claim, so there is --no federal question jurisdiction over (2).

Question 22 4 / 4 pts Defendant Darla lived in Nevada until five years ago, when she was transferred to California on a temporary basis, to handle a troubled franchise operation. She fully intended to return to Nevada after the issues were resolved, hopefully within six months. She rented a house in California for three months. Plaintiff Parker, a resident of Nevada, sued Darla in Nevada for breach of contract on a business deal. Parker filed the suit in state court. Can Darla remove the case to federal court? No, unless the amount in controversy exceeds $75,000. No, because the parties are not diverse. Yes, since Darla has moved to California. Yes, but only if the amount in controversy exceeds $75,000.

B is the best answer here, because --the federal court has no diversity jurisdiction in a breach of contract case based on state law --since Darla still intends to return to Nevada, and --Parker is a resident of Nevada. A is incorrect because --even if the amount in controversy exceeds $75,000, --the parties are not diverse, so --the case cannot be removed, and --in addition Darla cannot remove the case because --she is an in-state defendant. C is incorrect because --even though Darla has moved to California, she has not changed her domicile since she does not intend to stay there, so she is still a resident of Nevada, and --the parties are not diverse, since --Parker is also a resident of Nevada. --D is incorrect because --even if the amount in controversy is met, the federal court has no subject matter jurisdiction, since --the case involves a state law claim and --the parties are not diverse given that Darla intends to return to Nevada.

Question 24 4 / 4 pts Plaintiff Parris, a resident of Ohio, sued defendant Devin, a resident of New York, in federal court in the state of New York, asserting that Devin, while acting as executor of an estate, violated his fiduciary duties under New York law, resulting in the loss of $1 million. Parris's process server caused the complaint to be served on Devin's wife by leaving a copy with her at Devin's home, as allowed by the Federal Rules of Civil Procedure. State law required personal service on the defendant for the specific claims asserted in Parris's complaint. Which rule applies and is the service made by Parris proper? Yes, service is proper and the federal rule applies because the rules are not in conflict. Yes, service is proper and the federal rule applies because it applies and is valid. No, service is not proper because the state rule applies because the rules are in conflict. No, service is not proper because the state rule applies since the claim arises under state law.

B is the best answer under --the Hanna case, which held that --FRCP 4 applied, since it is --procedural and is valid. A is incorrect because --the rules are in conflict. C is incorrect because --service is proper under FRCP 4. D Is incorrect because --in deciding which rule applies, --the substantive nature of the claim is not relevant.

Question 5 6 / 6 pts Attorney from State A helped Client from State B to purchase property in State A. Attorney and Client corresponded regularly by mail as the deal occurred. Client failed to pay Attorney the agreed upon fees. As a result, Attorney sued Client in State A. Attorney then published a notice of the suit in a local State A newspaper, as permitted by State A statute. Client failed to answer and default was entered. After judgment was entered, Client received notification of the judgment at her regular address and then challenged the judgment as constitutionally defective due to lack of notice. How should the court rule? Notice by publication was adequate under the State A statute. Notice by publication was adequate under the U.S. Constitution. Notice is unconstitutional as Attorney could have tried certified mail. Notice is unconstitutional as personal delivery is required.

C is the best answer based on Mullane, --which requires diligent efforts at informing an absentee designed to accomplish notice, and --certified mail would be preferable to publication in an out of state newspaper. A and B are incorrect because --publication out of State is not reasonably designed to provide notice and --fails to satisfy constitutional due process under Mullane. D is incorrect because --certified mail would suffice, and personal delivery is not required.

Question 4 6 / 6 pts Carly, a native of New York, lived her entire life in New York until she went off to college in California. She intended to return to New York, but voted in California. Upon graduation, she decided to go to graduate school in Washington D.C. now planning to complete law school, then return to New York to take the bar exam and get a job. During her second year of law school, she hit a pedestrian who was a lifelong resident of the District of Columbia. The family of the deceased sued her for wrongful death for $1 million in the US District Court for the District of Columbia. Carly moved to dismiss the case for lack of subject matter jurisdiction. How should the court rule? Grant the motion, because the parties are not diverse. Grant the motion, because there is no federal question involved. Deny the motion, because diversity jurisdiction exists. Deny the motion, because supplemental jurisdiction is available.

C is the best answer because --Carly never established a new domicile in DC and --so remains a citizen of NY for purposes of --diversity analysis. A is incorrect because --the parties are diverse, since --Carly's domicile is in NY to which she intends to return. B is incorrect because --even though there is no federal question, --there is diversity jurisdiction available. D is incorrect because --supplemental jurisdiction is --not applicable here since --there is original jurisdiction.

Question 5 10 / 10 pts Rasha sued Kahn, who lived much of his time in an apartment in New York, where he had certified to the building that he resided, but frequently travelled abroad to Saudi Arabia. Rasha served the summons and complaint at Kahn's New York apartment where he had been staying for a month. Kahn's live in maid, age 42, accepted service. Kahn failed to answer the complaint based on his assumption that service on his maid was not valid. The court entered a default based on Rasha's request, and entered judgment against Kahn. Kahn then filed a motion to set aside the default claiming the service of the complaint was invalid. How should the court rule? Grant the motion because service on his maid is invalid. Grant the motion because Kahn did not reside in New York. Deny the motion because service was valid. Deny the motion because Kahn spent substantial time In Saudi Arabia

C is the best answer because --Kahn is regarded as a resident of New York, and --service at his place of residence upon his maid --is adequate, so that --service was valid and --the default should stand. A is incorrect because --service on his maid is valid. B is incorrect on the facts stated. D is incorrect because --only his place of residence is a valid place of service. Quiz Score: 40 out of 50

Question 16 4 / 4 pts Mel, a magician, lives in California, but performs several times a year in Georgia. While driving from the airport in Atlanta to a performance, Mel is in an auto accident with Pat. After Mel returns to California, Pat sues Mel in Georgia. The Georgia long arm statute provides that there is jurisdiction over those who (1) transact business in the state of Georgia or (2) commit a tortious act in the state of Georgia, not involving defamation. Mel files a motion to dismiss for lack of jurisdiction. Does the Georgia state court have jurisdiction over Mel? No, there is jurisdiction under the long arm statute, but it would violate due process. No, there is no jurisdiction under the long arm statute.! Yes, there is jurisdiction under the long arm statute, and it is consistent with due process. Yes, the state of Georgia has general jurisdiction over Mel.

C is the best answer because --Mel meets both (1) and (2) under the long arm statute and --his conduct involves minimum contacts with the state, from which the claim arises, so --specific jurisdiction is available, given that --he availed himself of the benefits of the state, and --it was foreseeable he might be haled into court in the state, and --arguably reasonable since he travels somewhat frequently, and --the plaintiff has an interest in a convenient forum, and --the state has an interest in protecting its residents. A is incorrect because --jurisdiction would not violate due process for the reasons stated. B is incorrect because --there is jurisdiction under the long arm statute. D is incorrect because --Mel's contacts do not seem pervasive enough for general jurisdiction.

Question 2 10 / 10 pts Pat alleges he suffered injury when Don drove his vehicle through a red light and struck Pat's car. Don moves for summary judgment on the basis of his own affidavit and those of five clergymen who witnessed the accident, all of whom say Pat went through the light. In response, Pat, counters with an affidavit from an alcoholic with three convictions for driving under the influence who also saw the accident, indicating that Pat's version of the facts is correct. How should the court rule? The court should grant the motion because the evidence from the alcoholic is not admissible. The court should grant the motion because the evidence from the clergymen is more credible than the evidence from the alcoholic. The court should deny the motion because an issue of credibility exists. The court should deny the motion because the evidence from the clergymen is not admissible.

C is the best answer because --courts cannot resolve a case on summary judgment where --an issue of credibility exists and --here, the clergymen and alcoholic disagree, creating an issue of credibility. --A is incorrect because --the alcoholic has personal knowledge of the accident, --so his statement will be admissible. --B is incorrect because --the court does not make --determinations of credibility in --ruling on summary judgment. --D is incorrect because --the testimony from the clergymen is --admissible, as they too were eyewitnesses.

Question 3 10 / 10 pts Zenith Insurance from New York initiates an interpleader action joining Ron from New York, Sam from New York and Sally from California, who all three have claims to the total proceeds of a $1 million Zenith insurance policy that covered harm occurring in a bus accident, after each of them suffered catastrophic injuries. Which of the following is correct? If Zenith relies on rule interpleader, the court has subject matter jurisdiction in the action. If Zenith relies on rule interpleader, the court has subject matter jurisdiction based on the amount in controversy. If Zenith relies on statutory interpleader, the court has subject matter jurisdiction. If Zenith relies on statutory interpleader, the court has no subject matter jurisdiction.

C is the best answer because --only minimal diversity between any two claimants is required and --Sally and Sam are diverse, and --only a $500 amount in controversy is required, which --is satisfied. A is incorrect because --rule interpleader requires complete diversity --between the stakeholder and all claimants so --there is no subject matter jurisdiction. --B is incorrect because --the parties are not diverse, since --Zenith, Ron and Sam are all from New York.

Question 5 6 / 6 pts Kids, Inc. is a Delaware Corporation with its headquarters in Ohio, operating an online toy store world-wide, using a toll free number to take orders, and marketing itself over the internet through pop-up advertisements on children's websites. Kids, Inc. does not have any retail outlets in Florida, but ships about 15% of its total sales into Florida annually over the last five years. Plaintiff Pam, a resident of Florida, is injured when she flips over in a car her parents purchased through the Kids, Inc. website in a Florida state court, alleging state law tort claims. Kids, Inc. files a motion to dismiss. Florida has a long arm statute extending personal jurisdiction to the extent permitted by due process. How should the court rule? Grant the motion because Kids, Inc.'s contacts with Florida do not relate to Pam's claim. Grant the motion because Kids, Inc. did not purposefully avail itself of the benefits of the state of Florida. Deny the motion because exercising jurisdiction over Kids, Inc. comports with traditional notions of fair play and substantial justice. Deny the motion because the Florida court has general jurisdiction over Kids, Inc.

C is the best answer because --the contacts of Kids Inc., and --the product shipped into the state, combined with --its internet sales and advertising, means\ --it is fair for a Florida court to assert jurisdiction over Kids, Inc. A is incorrect because --the internet related contacts of Kids Inc. with Florida --do relate to the claim being made by Pam. B is incorrect because --Kids Inc. is shipping a large percentage of its product into the state of Florida and --combined with its marketing, would most likely be found --to have purposefully availed itself of the benefits of doing business in the state. D is incorrect because --Kids Inc. is not present in Florida, since --it has no retail operations there and --its headquarters and state of incorporation are in other states. Quiz Score: 24 out of 30

Question 3 4 / 4 pts Plaintiff Petra sued defendant Danico, the manufacturer of a machine, for products liability after Petra was injured on the job. Petra filed the case in federal court in Arizona based on diversity. The federal rules of civil procedure do not require that Petra plead any defenses in her complaint. However, Arizona law requires that Petra allege that she was free from contributory negligence. Danico moves to dismiss based on Petra's failure to allege that she was free from contributory negligence. How should the court rule? Grant the motion because the Arizona law is on point. Grant the motion because the Arizona law is substantive. Deny the motion because the federal rule is on point and in conflict. Deny the motion because the federal rule is substantive.

C is the best answer because --the federal rule covers the point, and --conflicts with the state law about pleading, and --the federal rule is valid, and --will apply, under Hanna. A is incorrect because --although the Arizona law is on point, --where it conflicts with a federal procedural rule, --the federal procedural rule will apply. B is incorrect because --the state law will not be regarded as substantive, but --as a rule of pleading, --which is procedural, and --the motion would not be granted. D is incorrect because --the federal rule is a procedural rule.

Question 4 10 / 10 pts Paul from California sued Dillon from Arizona for breach of contract for $100,000 related to the sale of a business. In the same case, Dillon filed a claim against Paul for breach of the same contract. Dillon also asserted a second claim for breach of an earlier contract on an unrelated matter, seeking $50,000. Paul filed a motion to dismiss for lack of subject matter jurisdiction. Which of the following is correct? The court has subject matter jurisdiction over both of Dillon's claims. The court has subject matter jurisdiction over only Dillon's second claim. The court has subject matter jurisdiction over only Dillon's first claim. The court has no subject matter jurisdiction over either claim.

C is the best answer because --the first claim is a compulsory counterclaim, which is --part of the same transaction or occurrence, and --the court has original jurisdiction over the claim here because --it meets the amount in controversy requirement and --the parties are diverse. A is incorrect because --the court does not have original jurisdiction over Dillon's second claim, and --has no supplemental jurisdiction because --the claim does not meet the amount in controversy. C is incorrect for the same reason. D is incorrect because --the court has jurisdiction over Dillon's first claim.

Score for this quiz: 30 out of 50 Submitted Aug 24 at 2:06am This attempt took 10 minutes. Question 1 10 / 10 pts Marty sued Felix for breach of contract. Felix moved to dismiss for lack of personal jurisdiction. The motion was denied. Felix now moves to dismiss the complaint for insufficient service of process. What result? Granted, if service was not sufficient. Granted, if Felix was not personally served. Denied, because it was waived. Denied, but only if Felix had actual notice.

C is the best answer because --the motion for insufficient service of process is --waived if -it is not raised along with the --motion addressing personal jurisdiction. A is incorrect because --even if service was insufficient, --the argument was waived. B is incorrect because --even if Felix was not personally served --the argument was waived. --D is incorrect because --Felix's actual notice is not relevant to service.

Question 4 4 / 4 pts Plaintiff Peter was a resident of California. He was involved in an automobile accident with Defendant Darwin, a resident of Oregon. Darwin also owned property in California. The medical bills and damages to Peter's car exceeded $75,000. Peter filed the lawsuit in federal court in Oregon. After the lawsuit was filed, Darwin moved to California. Shortly before trial, Darwin moved to dismiss the case based on the lack of subject matter jurisdiction. How should the court rule? Dismiss the case because of Darwin's move to California. Dismiss the case because the court had no subject matter jurisdiction when the case was filed. Deny the motion because the parties were diverse when the case was filed. Deny the motion because Darwin should have filed it sooner.

C is the best answer because --the parties were diverse based on Darwin's domicile, --which was in Oregon, and --is not affected by his ownership of property in another state. A is incorrect because --a party's change of domicile after the case is filed --does not impact or defeat jurisdiction based on diversity. B is incorrect because --the court had subject matter jurisdiction --when the case was filed --based on diversity. --D is incorrect because --subject matter jurisdiction --is never waived

Question 10 4.5 / 4.5 pts Portia sued Ottie Autos for negligently inspecting her car resulting in an unanticipated acceleration that caused her severe injuries. Two years later, after the statute of limitations had expired, and after extensive discovery in the case, Portia filed a motion seeking to add a claim against Ottie Autos for product liability based on a manufacturing defect. The court should: Deny the motion because the statute of limitations has expired. Deny the motion because the claim cannot be joined in the same action. Grant the motion as long as Ottie Autos is not prejudiced. Grant the motion only if Portia can demonstrate mistake.

C is the best answer because leave to amend is liberally granted if the defendant is not prejudiced. A is incorrect because under the relation back doctrine, the amendment will relate back to the date the action was filed. B is incorrect because a plaintiff may join as many claims as she has against a defendant in a single action. D is incorrect because the plaintiff is not required to demonstrate a mistake in order to receive leave to amend.

Question 17 4.5 / 4.5 pts After a Subway train derailed, seventeen passengers were killed and 180 passengers were injured. Subway carried insurance for $20 million through Arrow Insurance. The passengers who were killed were from Delaware, Massachusetts and New Hampshire. Subway is located in Massachusetts. Arrow is located in Florida. If the survivors of the decedents seek to assert claim against the insurance policy issued by Arrow, assuming the amount in controversy is satisfied, which joinder device is appropriate? Only rule interpleader, because the parties are diverse. Only statutory interpleader, because the diversity requirement is met. Both statutory and rule interpleader are available. Neither statutory nor rule interpleader apply on these facts.

C is the best answer because rule interpleader requires that the stakeholder and claimants are diverse, which is the case here, and statutory interpleader requires only that any two claimants be diverse, which is also the case here. A is incorrect because statutory interpleader is also available. B is incorrect because rule interpleader is also available. D is incorrect because both statutory and rule interpleader are available.

Question 11 0 / 4.5 pts Matt (from New York) sued Frank (from Florida) seeking $70,000 alleging antitrust violations under federal law. Frank filed a motion to dismiss the case for improper venue. The motion was denied. Frank then filed a motion to dismiss for failure to state a claim, and due to lack of subject matter jurisdiction. How should the court rule? Grant the motion to dismiss for lack of subject matter jurisdiction and the motion to dismiss for failure to state a claim. Grant the motion for failure to state a claim, and deny the motion based on subject matter jurisdiction. Deny the motion to dismiss for failure to state a claim, and deny the motion based on lack of subject matter jurisdiction. Deny the motion to dismiss for failure to state a claim, but grant the motion due to lack of subject matter jurisdiction.

C is the best answer because the motion to dismiss for failure to state a claim was waived when Frank filed the motion for change of venue; the motion for subject matter jurisdiction is still viable, but the court has subject matter jurisdiction based on a federal question regarding antitrust law. A is incorrect because the motion to dismiss for failure to state a claim was waived and the court has subject matter jurisdiction based on the federal question. B is incorrect because the motion to dismiss for failure to state a claim was waived when Frank filed the motion for change of venue. D is incorrect because the court has subject matter jurisdiction based on the federal question.

Question 13 4.5 / 4.5 pts Ryan, on vacation from Utah, traveling in California was involved in an auto accident, and Plato, from California, a pedestrian bystander, was injured by flying debris. Plato was treated by Doctor, also a California resident, which exacerbated his condition. Plato sued both Ryan, for negligence, and Doctor, for medical malpractice. Doctor sought a medical examination of Plato. Ryan issued a request for discovery of Plato's medical records from treatment by Doctor. Plato's attorney should: Object that the records from treatment are not relevant to Ryan, but make Plato available for a medical examination. Produce the records from treatment by Doctor, but object to the request for the medical examination. Produce the records from treatment by Doctor and make Plato available for a medical examination. Object that the records from treatment are not relevant to Ryan and object to the request for the medical examination.

C is the best answer because the records are relevant to Plato's damages in the case against Ryan there is good cause for the medical examination since Plato is suing Doctor for medical malpractice. Therefore A is incorrect because the records are relevant. B is incorrect because there is good cause for the medical exam. D is incorrect because the records are relevant to Plato's damages in the case against Ryan there is good cause for the medical examination since Plato is suing Doctor for medical malpractice.

Question 11 0 / 4 pts Ben owns a New York jazz club, the "Blue Note," with a federally registered trademark. King, from Missouri, owns a small nightclub also called the Blue Note. King posted a webpage with a logo similar to Ben's trademark. King's site has info about shows at his Missouri club, and a phone number to call for tickets for pickup. Ben sued King for trademark infringement in New York. New York's long arm statute permits personal jurisdiction. Does New York have personal jurisdiction over King? Yes, as it was foreseeable that King's site could be viewed in New York. Yes, because King targeted the New York forum. No because King did not purposefully avail himself of the benefits of New York. No because King's contacts with New York are minimal and do not relate to the claim.

C is the best answer here because --exercising jurisdiction would violate due process. King did nothing to purposefully avail himself of the benefits of NY. --Applying the stream of commerce analogy, the court said --placing his site in the stream of commerce --did not suffice for PJ because --he did not actively seek to have NY residents go to his site, and --there was no continuous or systematic part of King's business in NY, --nor really any business presence in NY, except that the site could be accessed there. A is wrong because --it is the wrong foreseeability test - can he foresee being haled into NY? The court said no. B is incorrect because --there is no activity on the website that is aimed at NY. D is incorrect because --there are some minimal contacts and --they do relate to the claim, to the extent the site is viewable in NY and the similar trademark used on the site is the subject matter of the claim.

Question 25 0 / 4 pts Plaintiff Patty filed a lawsuit alleging sexual harassment against her employer, Mega Bank, in federal district court in the Central District of California. Defendant Mega Bank filed a motion to transfer under Section 1404 to the state of South Carolina, where the company is located. The court denied the defendant's motion. May Mega Bank move to dismiss for lack of personal jurisdiction? No, defendant waived the personal jurisdiction objection by filing the transfer motion. No, because defendant consented to jurisdiction by filing the motion to transfer. Yes, filing a transfer motion does not waive the personal jurisdiction objection. Maybe, as there is a split of authority on this issue.

C is the best answer here because --filing the motion to transfer is not a motion filed pursuant to Rule 12, so under 12(g)(2) it is not a motion under this rule, so --it does not waive personal jurisdiction and --the defendant CAN still file the motion. A is incorrect because --there is no waiver under Rule 12. B is incorrect because --filing the motion to transfer does not constitute consent to personal jurisdiction, --which can still be raised. D is incorrect because --there is no split on --whether a motion to transfer waives a personal jurisdiction objection.

Question 4 10 / 10 pts Pam's husband died in a plane crash. Ten survivors filed separate wrongful death cases against the Airline in separate states. In seven cases, the jury found that the Airline was not negligent. In three cases, the jury found that the Airline was negligent. Pam filed case number 11, seeking damages for wrongful death against Airline. Pam files a motion for summary judgment, seeking to use the determinations that the Airline was negligent in three cases to preclude the Airline from litigating the cases based on the doctrine of collateral estoppel. How should the court rule? Grant Pam's motion because the issue was previously actually litigated. Grant Pam's motion because the issue was previously necessarily determined. Deny Pam's motion due to the inconsistent verdicts. Deny Pam's motion because the prior determination was not essential to the judgments.

C is the best answer under Parklane, because --it is not fair for Pam to assert collateral estoppel when --there are inconsistent judgments. --A is incorrect, because --even though the issue was actually litigated, --it would not be fair to --use collateral estoppel due to --the inconsistent judgments. --B is incorrect because --even though the issue was necessarily decided --it would not be fair to allow use of --collateral estoppel here due to the inconsistent judgments.

Question 5 4.5 / 4.5 pts Raven, a film star residing in New York, sued Quartz Pictures, a movie studio located in California, for defamation and sought both damages of $1 million and preliminary and permanent injunctive relief after Quartz employees allegedly released false information concerning Raven's most recent visit to a rehab clinic to a tabloid publication. Raven has filed her case in federal court in Southern California and Quartz made a timely demand for a jury trial. The court granted Raven's request for a preliminary injunction. At trial, The jury should determine all factual issues and address all remedies. The court should address injunctive relief first, and then the jury should determine damages. The jury should determine the facts and award damages, and then the court should address injunctive relief. The court should determine all issues, since Raven is seeking injunctive relief.

C is the best answer under the Beacon case which held that legal claims should be tried first to the jury and the court should then address equitable claims. A is incorrect because the jury does not address equitable claims. B is incorrect because the case should be tried to the jury first. D is incorrect because Quartz made a timely demand for jury trial and has a right to jury trial on legal claims under the 7th Amendment.

Question 9 4 / 4 pts Mr. P sues Ms. D in California for breach of contract. Ms. D owns property in California that is unrelated to the contract and now lives in Nevada. Mr. P gives notice by substituted service and attaches Ms. D's California property. Does the court have personal jurisdiction? No, because Ms. D is not present in California. No, because Ms. D's property ownership is not enough for the court to exercise jurisdiction. Yes, if Ms. D's property is attached, but only to the extent of the value of the property. Yes, if Ms. D's property is attached, to the extent of all her assets.

C is the best answer, based on --quasi in rem jurisdiction, based on --her property, and --the extent of the judgment would extend only to --the value of the property. A is wrong because --though there would not be in personam PJ based on Ms. D's presence, --there is PJ over Ms. D based on --the presence of her property. --B is wrong because --her ownership of property is --enough of an anchor for PJ. D is wrong because --the court has PJ only over the property, so not to the extent of all of Ms. D's assets.

Score for this quiz: 30 out of 30 Submitted Jul 12 at 12:47am This attempt took 14 minutes. Question 1 6 / 6 pts While driving in Nevada on his first visit to the state, David's car collided with a car driven by Paula. David is a citizen of California. Paula is a citizen of Nevada. Paula sued David in Nevada state court for the damages to her car and personal injuries incurred in the accident. Is David constitutionally subject to in personam jurisdiction in Nevada? Yes, the court has specific jurisdiction over David based on his contacts with Nevada. No, David is not subject to in personam jurisdiction based on his single contact with Nevada. No, because there is no nexus between the claim and David's contact with Nevada. Yes, the court has in rem jurisdiction over David based on his contacts with Nevada.

Correct! A is the best answer because --David's presence in Nevada is a contact that will support the exercise of in personam jurisdiction over him in the State of Nevada. B is incorrect because --a single contact is sufficient, --where the contact is related to the claim. C is incorrect because --there is a nexus between the claim and David's contact with the state. --D is incorrect because --there is no indication that David has any property in the state which would support in rem jurisdiction.

Question 5 6 / 6 pts Martha lives in Iowa on a farm. Her former business partner Samuel believes that Martha owes him money based on a failed business venture in Delaware. Samuel sued Martha in Iowa, and had her served with the suit while she was in the Hamptons in New York visiting family and friends. Do the Iowa courts have personal jurisdiction over Martha? Yes, because Martha lives in Iowa. Yes, but only if Martha's contacts with Iowa somehow relate to the claim. No, because she was not served in Iowa. No, because the venture was in Delaware.

Correct! A is the best answer because --a court has personal jurisdiction over a party where --a party is present in the state based on their domicile, and --Martha is present in Iowa because --she lives there on a farm, so that --Iowa is her domicile. B is incorrect because --where a person is present, the court has jurisdiction over the person based on presence and analysis of contacts and their relationship to the claim is irrelevant. C is incorrect because --it does not matter where a party is served, if the party is present in the state as Martha is here, the court will have personal jurisdiction. D is incorrect because --it does not matter where the venture occurred, and --the court will have jurisdiction based on the party's presence. Quiz Score: 30 out of 30--

Question 3 6 / 6 pts Wilma, a resident of Ohio, alleges that she entered into an oral agreement with Fred, a resident of Montana, one evening when she met him in a bar in Ohio. Pursuant to the deal, Wilma quit her teaching job to work for Fred in his business in Ohio, in exchange for which Fred agreed to support Wilma for the rest of her life. Fred visited Ohio occasionally on business, and purchased a condo in Ohio with Wilma, where Fred sometimes stayed. Fred later stopped supporting Wilma and returned to Montana and Wilma sued Fred in Ohio to enforce the agreement. Ohio has a Long Arm Statute which subjects defendants to jurisdiction based upon causing torts within Ohio, entering contracts in Ohio and engaging in business dealings in Ohio, among other things. Fred challenged the Ohio State Court's jurisdiction over him. In ruling on the motion, what is the Ohio State Court most likely to find regarding the application of the Long Arm Statute? The Long Arm Statute applies to allow jurisdiction over Fred based on his tortious conduct in Ohio. The Long Arm Statute applies to allow jurisdiction over Fred based on his contract with Wilma. The Long Arm Statute does not allow jurisdiction because Fred's conduct does not relate to his contacts with the forum. The Long Arm Statute does not allow jurisdiction because Fred's business dealings in Ohio were only occasional.

Correct! B is the best answer based on the Burger King case, which held that --entering into a contract followed by a substantial and continuous relationship sufficed for purposes of jurisdiction. A is incorrect because --the suit is for enforcement of a contract, not based on tortious conduct. C is incorrect because --the suit does relate to Fred's contacts with the forum. D is incorrect because --of his business Fred has an ongoing business in Ohio for which Wilma worked pursuant to the deal, and Fred also visited Ohio.

Question 2 6 / 6 pts Pistol Co., a New York corporation with its manufacturing plant and executive offices in New Jersey, makes guns and sells them through distributors in New Jersey, New York and Connecticut, which in turn sell to retailers only in the same three states. While on a hunting trip to Connecticut, Vic, a resident of California, buys one of Pistol's guns from a Connecticut retailer. When Vic fires the gun upon his return to California, it explodes in his face causing serious injury. If Vic sues Pistol in a California state court for his injuries, the court Has jurisdiction over Pistol because Pistol is "present" in California. Has jurisdiction over Pistol because Pistol is doing business in California. Has no jurisdiction over Pistol because the gun that exploded got to California as a result of Vic's actions, and not through any act by Pistol. Has no jurisdiction over Pistol unless another gun owner also brought a Pistol gun into California.

Correct! C is the best answer because -- there is no indication that Pistol put its product into the stream of commerce with any knowledge or intent that it would reach California. World Wide Volkswagen barred jurisdiction in situations where the product reaches the state through unilateral activity of the consumer. A is incorrect because --there is no indication that the company is present in California, since it does not have any operations there. B is incorrect because --there is no indication Pistol is doing business in California. D is incorrect because --even if another gun owner brought a gun into California, that unilateral activity would not be enough for jurisdiction without some knowledge or intent regarding Pistol products being in the state.

Question 12 0 / 4.5 pts Elvin sued Mack claiming that Mack made false statements accusing Elvin of blackmail and extortion. Elvin alleged that Mack, Wanda, who is Mack's wife, and Bart, who is Mack's brother, all heard the alleged slanderous statements. Mack moved for summary judgment. In support of the motion, Mack denied making the alleged statements and Wanda and Bart each submitted affidavits in which they denied hearing any of the alleged statements. Elvin opposed the motion with an affidavit of his own stating that although he was not present, he believed that Wanda and Bart had heard the statements. How should the court rule? Deny the motion because there is a genuine issue of material fact. Deny the motion to allow Elvin to cross-examine Mack and Wanda at trial. Delay ruling on the motion so that Elvin can take the depositions of Wanda and Bart. Grant the motion, because there is no genuine issue of material fact.

D is the best answer as Elvin has failed to raise any issue of material fact and did not request any delay for purposes of further discovery. A is incorrect because Elvin's evidence does not show that there are any witnesses to testify based on personal knowledge in support of his claims. B is incorrect because Elvin has the opportunity to take the testimony of Wanda and Bart under oath, but did not do so, and did not request a delay for that purpose. C is incorrect because Elvin did not make that request.

Question 7 4.5 / 4.5 pts Jill, from Oregon, and Ken, a California resident, got into an automobile accident, injuring both of them seriously, and also damaging both of their cars. Jill sued Ken, alleging that his negligence caused the accident and seeking damages exceeding $200,000. Ken filed a counterclaim for personal injury. He did not seek recovery for the damages to his car. The merits of all claims were resolved through a trial of the action. About six months later, Ken filed a second action seeking recovery for the damages to his car arising out of the same accident. Which of the following are correct? I. Ken's claim in the second case was a compulsory counterclaim in the first case. II. Ken's claim in the second case is barred by the doctrine of collateral estoppel. III. Ken's claim in the second case is barred by the doctrine of res judicata. All of the above. Only I and II. Only II and III. Only I and III.

D is the best answer because --a claim arising out of the same transaction or occurrence is --a compulsory counterclaim, and is --barred by the doctrine of res judicata because --it could have been litigated in the first case, and --a party cannot split his claim into two parts; the doctrine of collateral estoppel would not apply because --the issue has not been previously actually litigated as required for that doctrine.

Question 2 6 / 6 pts Landon, a resident of Michigan, rented a tractor from Agri Business, a New York, company. Pursuant to the terms of the lease, Landon agreed Art, an agent for service of process, would be appointed to receive service in New York, and that jurisdiction would be proper in New York. Landon had never been to New York. When Landon defaulted in payment due under the lease, Agri Business sued Landon in New York. Does the New York court have personal jurisdiction over Landon? No, because Landon had never been to New York. No, because Landon resides in Michigan. Yes, only if Landon mailed payments on the lease in New York. Yes, because Landon consented to jurisdiction in New York.

D is the best answer because --a party can consent to jurisdiction --through a contract, which is what Landon did in this case. A is incorrect because --it does not matter whether he has ever been to New York, given his consent to jurisdiction. B is incorrect because --it does not matter that he resides in Michigan, given his consent to jurisdiction in New York. C is incorrect, because --although payments would support an argument for jurisdiction based on minimum contacts, that is not necessary since --he consented to jurisdiction.

Question 5 0 / 10 pts Plato sued Diego for personal injury based on an automobile accident. The action went to trial and Plato prevailed. Three months later, Diego, very upset, consulted a new lawyer regarding whether he could bring a claim against Plato arising out of the same automobile accident. The best advice is that: Diego may bring his claim against Plato, as long as there is a good faith basis in fact and law for the claim. Diego may bring his claim as it will relate back to the filing of Plato's case against Diego. Diego's claim against Plato can be asserted unless it is barred by the applicable statute of limitations. Diego cannot bring his claim against Plato, as he should have asserted it at the time Plato sued him.

D is the best answer because --the claim is --a compulsory counterclaim which is --barred if it is not asserted in the original case. B is incorrect because --the relation back doctrine applies to --amendment of pleadings within a case, not --to allow parties to bring a separate action. C is incorrect because --the claim cannot be asserted even if --it is not barred by the statute of limitations, since --it is a compulsory counterclaim and --was not brought in the original case. Quiz Score: 30 out of 50

Question 3 0 / 10 pts A group of 190 past and present computer programmers for WebCo filed a class action lawsuit against the company in federal court, claiming they were improperly classified as exempt. Wanda works at WebCo. She sent a memo to WebCo's President stating she believed that the programmers were not exempt and should be paid overtime. WebCo should: Provide Wanda's name and the memo to the plaintiffs in its initial disclosures. Provide Wanda's name, but not the memo, in its initial disclosures. Provide the memo, but do not identify Wanda specifically, in its initial disclosures. Not provide Wanda's name, nor produce the memo, in its initial disclosures.

D is the best answer because --the initial disclosures --require disclosure of --those who will support the --claims or defenses, and --Wanda and her memorandum appear to --support the plaintiffs' claims. Therefore A, B and C are incorrect.

Question 4 0 / 10 pts On February 1, two contractors file a complaint in federal court alleging that a contractor negligently constructed their home causing them injuries. On February 15 of the same year, the contractor filed and served a motion to dismiss for failure to state a claim, arguing that under applicable state law the contractor is immune. On March 15, the homeowners sought to amend their complaint to allege that the contractor was reckless in constructing their home, a claim to which the contractor would not be immune. If the contractor objects to the amendment, how should the court rule? The court's permission is not required for the amendment. The court should not grant permission for the amendment. The amendment should not be permitted due to prejudice against the contractor. The amendment should be permitted by the court.

D is the best answer because --there is no indication of bad faith on the part of --the homeowners and --no prejudice to the contractor. A is incorrect because --more than 21 days have passed since --the motion to dismiss. --B is incorrect because --there is --no indication of bad faith on the part of --the homeowners and --no prejudice to the contractor. C is incorrect because --there is no prejudice to the contractor.

Question 10 4 / 4 pts For several years US Tire, a Michigan Corp., purchased large quantities of valves from Valves, Inc., a Michigan Corp. UST put the valves into its tires and sold the tires to a distributor, Delco, located in Illinois, which sold the tires all over the country, including Indiana, where a tire exploded injuring Plaintiff. Plaintiff sued UST, Valves, and Delco in Indiana. Valves has no other contacts in Indiana. Does the Indiana court have personal jurisdiction over Valves, Inc.? Yes, because UST's contacts with Indiana are sufficient for personal jurisdiction over Valves. No, because it would not be reasonable for the court to exercise jurisdiction over Valves. Yes, because Valves purposefully availed itself of the benefits of doing business in Indiana, under O'Connor's view. No under O'Connor's view, because Valves does not have the required minimum contacts with Indiana.

D is the best answer because --there is reasonableness here, but --the contacts are not sufficient. Certainly, under O'Connor's test, --there are no plus factors. --Under Brennan's test, --it is not even clear that --there is "mere awareness" A is incorrect, because --it is not UST's contacts that the court will look at, but --rather Valves, Inc's contacts that should be measured. Also, O'Connor would require that --there be some plus factors, which are missing here. Also, O'Connor's stream of commerce plus test does not appear to be met here, in that --Valves does not seem to have marketed/advertised or --designed its product for the market. Finally her test has not been applied in the 9th Cir. Where Cal if located. B is incorrect because --it would probably be reasonable to exercise jurisdiction here, just looking at the reasonableness factors - but --contacts are a problem. C is incorrect because --there is no indication that Valves --purposefully availed itself, since --none of the plus factors are present under O'Conner's view. However, we might also consider how would Brennan's test apply here? As far as the contacts/purposeful availment part of the test, since --defendant received benefit from sales in the state, as well as --indirect benefits under state law, --Brennan says there would be a basis for jurisdiction. But unlike Asahi, jurisdiction would be more reasonable here, because, --there is a stronger state interest (Indiana law could apply, --Indiana resident still in the case, unlike Zurcher), and plaintiff interest (like WWVW) -

Question 2 4 / 4 pts Plaintiff Pam, from Maryland, filed suit against defendant Daryl, from the Southern District of New York, and defendant Donna, from the Eastern District of New York. The events involved occurred in the Eastern District of New York. Subject matter jurisdiction is based on diversity. Venue is proper Only in the Southern District of New York. Only in the Eastern District of New York. Only in Maryland. In both the Southern and Eastern Districts of New York.

D is the best answer because --venue is proper in any district where --any defendant resides so long as --all defendants reside in the state containing that district. Here --both Daryl and Donna are from the state of New York, so --venue is proper in either district. Accordingly, A and B are incorrect. C is incorrect because --the plaintiff's state of residence is --not a proper venue under 28 USC 1391.

Question 15 4.5 / 4.5 pts Sol, a resident of Nevada, was seriously injured when Calvin, an employee of Contractor, dropped a wrench from 12 floors above while he was working in a building under construction. Sol, who had been passing by the construction site when he was hit by the falling wrench in the shoulder, incurred damages of $90,000, including loss of income as a chef. Sol filed claims against 1) Contractor, a California corporation contracted to complete construction of the building, for negligence based on vicarious liability; and 2) Calvin, a California resident, for negligence. Contractor moved to dismiss the case on the grounds of improper joinder. The court should: Grant the motion, because Calvin's liability is not derivative so joinder is improper. Grant the motion, because the claims do not arise from the same transaction or occurrence. Deny the motion, because the necessary supplemental jurisdiction exists. Deny the motion, because the claims share a logical relation.

D is the best answer because the claims involve the same occurrence, and will involve much of the same evidence, so joinder is proper. A is incorrect because joinder here is based on the joinder of defendants, not impleader, and the rules permit joinder of defendants when the claims share a common question of law or fact and arise from the same occurrence. B is incorrect on the facts. C is incorrect because supplemental jurisdiction is not relevant to this joinder claim, and at any rate, would not be needed because there is original jurisdiction for each claim.

Question 18 0 / 4.5 pts Ivan, a Carolina resident, was stopped by Metro Police pursuant to a new stop and frisk policy that was highly controversial. A scuffle with the Metro Police followed and Ivan suffered injuries at the hands of the police involved. Ivan sued the Metro Police for violation of his federal civil rights. Metro Police is a large urban police department in Carolina with over 9,000 officers. The Carolina Peace Officers' Association (CPOA) sought to intervene in the action. The CPOA is a statewide voluntary association in the state of Carolina that has advocated in favor of the stop and frisk policy, in order to protect public safety. Ivan objected to the request to intervene. What is the most likely result on the joinder issue? CPOA can intervene as of right because its interests will be impaired otherwise. The court may permit intervention only if Metro's representation is not adequate. CPOA cannot intervene because the court has no subject matter jurisdiction. CPOA cannot intervene unless it has a claim or defense related to the action.

D is the best answer because the court has discretion to allow intervention where a party has a claim or defense that involves a common question of law or fact with the pending action. A is incorrect, because there is no specific interest of the CPOA that would be impaired as a result of a decision in Ivan's case. B is incorrect because if Metro's representation is not adequate that would support intervention as of right, not permissive intervention which is available if the intervening party has a claim or defense that shares a common question of law or fact with the action. C is incorrect because the court will have subject matter jurisdiction based on the federal question, given this is a federal civil rights claim.

Question 14 4 / 4 pts Plaintiff Penny, a resident of Michigan, was driving in Indiana when she got into a car accident with Defendant Daryl, a resident of Kentucky. Penny sued Daryl in state court in Michigan. Regarding personal jurisdiction, the Michigan court has: Specific jurisdiction over Daryl based on minimum contacts. General jurisdiction over Daryl based on his domicile. Jurisdiction based on diversity, assuming the amount in controversy is met. No personal jurisdiction over Daryl.

D is the best answer, because --Daryl has no contacts with Michigan and --therefore no specific jurisdiction there, and --no domicile in Michigan and --therefore no general jurisdiction there. A is incorrect because --specific jurisdiction requires contacts with the state related to the claim, and --none are present. B is incorrect because --general jurisdiction requires domicile or presence, which is not the case here. C is incorrect because --the question asks about personal jurisdiction and --the answer relates to subject matter jurisdiction.

Score for this quiz: 12 out of 30 Submitted Aug 9 at 12:37am This attempt took 14 minutes. Question 1 0 / 6 pts Suppose the federal court makes a judicial rule indicating that there is no duty in a particular situation involving negligence by a corporation, while state law creates such a duty. In a subsequent case alleging the negligence of a corporation, in which the issue regarding the corporation's duty is raised in connection with the negligence claim, should the federal court apply the federal judicial law or the state law? The court can apply the federal judicial law because it is valid. The court should apply the federal judicial law assuming that it is on point. The court should apply the state law only if jurisdiction for the case is based on diversity. The court should apply the state law because the federal law is not valid.

D is the correct answer because --federal courts cannot create federal substantive common law. A is incorrect because --the federal judicial law is not valid - --federal judicial courts have no power to make substantive federal laws (Erie). B is wrong because --even assuming the federal law is on point, --there is no power to make the law, so --it should not be applied. C is wrong because, --even if the claim is a supplemental claim to a federal question, --the court would be able to apply the state law.

Question 14 0 / 4.5 pts Plaintiffs filed a shareholders' derivative action requiring the filing of a bond by plaintiffs in order to proceed. Defendants moved to require the posting of a bond. The court denied the motion. Defendant appealed. Should the collateral order doctrine apply to allow appellate review? Yes, because the right is separable from the rights asserted in the action and review would effectively be denied. Yes, because the right to require a bond is a prerequisite to the shareholders' derivative suit and the rights are related. No, because the bond requirement is not collateral to the shareholders' rights asserted in the action. No, because the appeal on the bond requirement can be deferred until the case as a whole is adjudicated.

The correct answer here is A. This is hypothetical is similar to the Cohen case at page 1003. Court held that court has appellate jurisdiction. B is incorrect because it is not the test and the bond right is collateral to the shareholders derivative rights. C is incorrect because it is collateral. D is incorrect because if the appeal on the bond were deferred until the case as a whole is adjudicated, then it would be meaningless.


Ensembles d'études connexes

English Composition 2 Lesson 2 Quiz

View Set

PrepU Ch. 34-35 Suicide and Crisis Intervention

View Set

Cultural Competence - Chapter 21

View Set